SlideShare ist ein Scribd-Unternehmen logo
1 von 68
Downloaden Sie, um offline zu lesen
 
 
APTITUDE:
Time and Work
1. A can do a work in 15 days and B in 20 days. If they work on it together for 4 days, then the
fraction of the work that is left is :
A.
1
4
B.
1
10
C.
7
15
D.
8
15
Answer: Option D
Explanation:
A's 1 day's work =
1
;
15
B's 1 day's work =
1
;
20
(A + B)'s 1 day's work =
1
+
1
=
7
.
15 20 60
(A + B)'s 4 day's work =
7
x 4 =
7
.
60 15
Therefore, Remaining work = 1 -
7
=
8
.
15 15
2. A can lay railway track between two given stations in 16 days and B can do the same job in
12 days. With help of C, they did the job in 4 days only. Then, C alone can do the job in:
A. 9
1
days
5
B. 9
2
days
5
C. 9
3
days
5
D. 10
Answer: Option C
Explanation:(A + B + C)'s 1 day's work =
1
,
4
 
 
A's 1 day's work =
1
,
16
B's 1 day's work =
1
.
12
C's 1 day's work =
1
-
1
+
1
=
1
-
7
=
5
.
4 16 12 4 48 48
So, C alone can do the work in
48
= 9
3
days.
5 5
3. A can do a piece of work in 4 hours; B and C together can do it in 3 hours, while A and C
together can do it in 2 hours. How long will B alone take to do it?
A. 8 hours B. 10 hours
C. 12 hours D. 24 hours
Answer: Option C
Explanation:
A's 1 hour's work =
1
;
4
(B + C)'s 1 hour's work =
1
;
3
(A + C)'s 1 hour's work =
1
.
2
(A + B + C)'s 1 hour's work =
1
+
1
=
7
.
4 3 12
B's 1 hour's work =
7
-
1
=
1
.
12 2 12
B alone will take 12 hours to do the work.
4. If 6 men and 8 boys can do a piece of work in 10 days while 26 men and 48 boys can do the
same in 2 days, the time taken by 15 men and 20 boys in doing the same type of work will
be:
A. 4 days B. 5 days
C. 6 days D. 7 days
 
 
Answer: Option A
Explanation:
Let 1 man's 1 day's work = x and 1 boy's 1 day's work = y.
Then, 6x + 8y =
1
and 26x + 48y =
1
.
10 2
Solving these two equations, we get : x =
1
and y =
1
.
100 200
(15 men + 20 boy)'s 1 day's work =
15
+
20
=
1
.
100 200 4
15 men and 20 boys can do the work in 4 days.
--------------------------------------------------------
Time and Distance
5. An aeroplane covers a certain distance at a speed of 240 kmph in 5 hours. To cover the
same distance in 1 hours, it must travel at a speed of:
A. 300 kmph B. 360 kmph
C. 600 kmph D. 720 kmph
Answer: Option D
Explanation:
Distance = (240 x 5) = 1200 km.
Speed = Distance/Time
Speed = 1200/(5/3) km/hr. [We can write 1 hours as 5/3 hours]
Required speed = 1200 x
3
km/hr
= 720 km/hr.
5
 
 
6. If a person walks at 14 km/hr instead of 10 km/hr, he would have walked 20 km more. The
actual distance travelled by him is:
A. 50 km B. 56 km
C. 70 km D. 80 km
Answer: Option A
Explanation:
Let the actual distance travelled be x km.
Then,
x
=
x + 20
10 14
14x = 10x + 200
4x = 200
x = 50 km.
Simple Interest
7. A sum of Rs. 12,500 amounts to Rs. 15,500 in 4 years at the rate of simple interest. What is
the rate of interest?
A. 3% B. 4%
C. 5% D. 6%
E. None of these
Answer: Option D
 
 
Explanation:
S.I. = Rs. (15500 - 12500) = Rs. 3000.
Rate =
100 x 3000
%
= 6%
12500 x 4
8. A sum of money at simple interest amounts to Rs. 815 in 3 years and to Rs. 854 in 4 years.
The sum is:
A. Rs. 650 B. Rs. 690
C. Rs. 698 D. Rs. 700
Answer: Option C
Explanation:
S.I. for 1 year = Rs. (854 - 815) = Rs. 39.
S.I. for 3 years = Rs.(39 x 3) = Rs. 117.
Principal = Rs. (815 - 117) = Rs. 698.
9. A man took loan from a bank at the rate of 12% p.a. simple interest. After 3 years he had to
pay Rs. 5400 interest only for the period. The principal amount borrowed by him was:
A. Rs. 2000 B. Rs. 10,000
C. Rs. 15,000 D. Rs. 20,000
Answer: Option C
Explanation:
Principal = Rs.
100 x 5400
= Rs. 15000.
12 x 3
 
 
10. What will be the ratio of simple interest earned by certain amount at the same rate of
interest for 6 years and that for 9 years?
A. 1 : 3 B. 1 : 4
C. 2 : 3 D. Data inadequate
E. None of these
Answer: Option C
Explanation:
Let the principal be P and rate of interest be R%.
Required ratio =
P x R x 6
100
=
6PR
=
6
= 2 : 3.
P x R x 9
100
9PR 9
Profit and Loss
11. Alfred buys an old scooter for Rs. 4700 and spends Rs. 800 on its repairs. If he sells the
scooter for Rs. 5800, his gain percent is:
A. 4
4
%
7
B. 5
5
%
11
C. 10% D. 12%
Answer: Option B
Explanation:
Cost Price (C.P.) = Rs. (4700 + 800) = Rs. 5500.
Selling Price (S.P.) = Rs. 5800.
 
 
Gain = (S.P.) - (C.P.) = Rs.(5800 - 5500) = Rs. 300.
Gain % =
300
x 100
%
= 5
5
%
5500 11
12. If selling price is doubled, the profit triples. Find the profit percent.
Answer: Option B
Explanation:
Let C.P. be Rs. x and S.P. be Rs. y.
Then, 3(y - x) = (2y - x) y = 2x.
Profit = Rs. (y - x) = Rs. (2x - x) = Rs. x.
Profit % =
x
x 100
% = 100%x
13. A vendor bought toffees at 6 for a rupee. How many for a rupee must he sell to gain 20%?
A. 3 B. 4
C. 5 D. 6
Answer: Option C
Explanation:
C.P. of 6 toffees = Re. 1
S.P. of 6 toffees = 120% of Re. 1 = Rs.
6
5
For Rs.
6
, toffees sold = 6.
5
For Re. 1, toffees sold = 6 x
5
= 5.
6
 
 
14. A man buys a cycle for Rs. 1400 and sells it at a loss of 15%. What is the selling price of the
cycle?
A. Rs. 1090 B. Rs. 1160
C. Rs. 1190 D. Rs. 1202
Answer: Option C
Explanation:
S.P. = 85% of Rs. 1400 = Rs.
85
x 1400 = Rs. 1190
100
Percentage
15. A batsman scored 110 runs which included 3 boundaries and 8 sixes. What percent of his
total score did he make by running between the wickets?
A. 45% B. 45
5
%
11
C. 54
6
%
11
D. 55%
Answer: Option B
Explanation:
Number of runs made by running = 110 - (3 x 4 + 8 x 6)
= 110 - (60)
= 50.
Required percentage =
50
x 100
% = 45
5
%
110 11
 
 
16. If 20% of a = b, then b% of 20 is the same as:
A. 4% of a B. 5% of a
C. 20% of a D. None of these
Answer: Option A
Explanation:
20% of a = b
20
a = b.
100
b% of 20 =
b
x 20 =
20
a x
1
x 20 =
4
a = 4% of a.
100 100 100 100
17. In an election between two candidates, one got 55% of the total valid votes, 20% of the
votes were invalid. If the total number of votes was 7500, the number of valid votes that the
other candidate got, was:
A. 2700 B. 2900
C. 3000 D. 3100
Answer: Option A
Explanation:
Number of valid votes = 80% of 7500 = 6000.
Valid votes polled by other candidate = 45% of 6000
=
45
x 6000 = 2700.
100
18. Three candidates contested an election and received 1136, 7636 and 11628 votes
respectively. What percentage of the total votes did the winning candidate get?
 
 
A. 57% B. 60%
C. 65% D. 90%
Answer: Option A
Explanation:
Total number of votes polled = (1136 + 7636 + 11628) = 20400.
Required percentage =
11628
x 100
% = 57%.20400
19. The population of a town increased from 1,75,000 to 2,62,500 in a decade. The average
percent increase of population per year is:
A. 4.37% B. 5%
C. 6% D. 8.75%
Answer: Option B
Explanation:
Increase in 10 years = (262500 - 175000) = 87500.
Increase% =
87500
x 100
% = 50%.175000
Required average =
50
% = 5%.10
Average
20. In the first 10 overs of a cricket game, the run rate was only 3.2. What should be the run rate
in the remaining 40 overs to reach the target of 282 runs?
 
 
A. 6.25 B. 6.5
C. 6.75 D. 7
Answer: Option A
Explanation:
Required run rate =
282 - (3.2 x 10)
=
250
= 6.25
40 40
21. A family consists of two grandparents, two parents and three grandchildren. The average
age of the grandparents is 67 years, that of the parents is 35 years and that of the
grandchildren is 6 years. What is the average age of the family?
A. 28
4
years
7
B. 31
5
years
7
C. 32
1
years
7
D. None of these
Answer: Option B
Explanation:
Required average =
67 x 2 +
35 x 2 + 6
x 3
2 + 2 + 3
=
134 + 70
+ 18
7
=
222
7
= 31 5 years.
 
 
7
22. A grocer has a sale of Rs. 6435, Rs. 6927, Rs. 6855, Rs. 7230 and Rs. 6562 for 5
consecutive months. How much sale must he have in the sixth month so
that he gets an average sale of Rs. 6500?
A. Rs. 4991 B. Rs. 5991
C. Rs. 6001 D. Rs. 6991
Answer: Option A
Explanation:
Total sale for 5 months = Rs. (6435 + 6927 + 6855 + 7230 + 6562) = Rs. 34009.
Required sale = Rs. [ (6500 x 6) - 34009 ]
= Rs. (39000 - 34009)
= Rs. 4991.
23. he average weight of 8 person's increases by 2.5 kg when a new person comes in place of
one of them weighing 65 kg. What might be the weight of the new person?
A. 76 kg B. 76.5 kg
C. 85 kg D. Data inadequate
E. None of these
Answer: Option C
Explanation:
Total weight increased = (8 x 2.5) kg = 20 kg.
Weight of new person = (65 + 20) kg = 85 kg.
 
 
24. The average age of husband, wife and their child 3 years ago was 27 years and that of wife
and the child 5 years ago was 20 years. The present age of the husband is:
A. 35 years B. 40 years
C. 50 years D. None of these
Answer: Option B
Explanation:
Sum of the present ages of husband, wife and child = (27 x 3 + 3 x 3) years = 90 years.
Sum of the present ages of wife and child = (20 x 2 + 5 x 2) years = 50 years.
Husband's present age = (90 - 50) years = 40 years.
25. The average weight of 16 boys in a class is 50.25 kg and that of the remaining 8 boys is
45.15 kg. Find the average weights of all the boys in the class.
A. 47.55 kg B. 48 kg
C. 48.55 kg D. 49.25 kg
Answer: Option C
Explanation:
Required average =
50.25 x 16 + 45.15 x 8
16 + 8
=
804 + 361.20
24
=
1165.20
24
= 48.55
 
 
26. If the average marks of three batches of 55, 60 and 45 students respectively is 50, 55, 60,
then the average marks of all the students is:
A. 53.33 B. 54.68
C. 55 D. None of these
Answer: Option B
Explanation:
Required average =
55 x 50 + 60 x 55 + 45 x 60
55 + 60 + 45
=
2750 + 3300 + 2700
160
=
8750
160
Decimal Fraction
27. Evaluate :
(2.39)2
- (1.61)2
2.39 - 1.61
A. 2 B. 4
C. 6 D. 8
Answer: Option B
Explanation:
Given Expression =
a2
- b2
=
(a + b)(a - b)
= (a + b) = (2.39 + 1.61) = 4.
a - b (a - b)
 
 
28. What decimal of an hour is a second ?
A. .0025 B. .0256
C. .00027 D. .000126
Answer: Option C
Explanation:
Required decimal =
1
=
1
= .00027
60 x 60 3600
29. The value of
0.1 x 0.1 x 0.1 + 0.02 x 0.02 x 0.02
is:
0.2 x 0.2 x 0.2 + 0.04 x 0.04 x 0.04
A. 0.0125 B. 0.125
C. 0.25 D. 0.5
Answer: Option B
Explanation:
Given expression =
(0.1)3
+ (0.02)3
=
1
= 0.125
23
[(0.1)3
+ (0.02)3
] 8
30.
4.2 x 4.2 - 1.9 x 1.9
is equal to:
2.3 x 6.1
A. 0.5 B. 1.0
C. 20 D. 22
Answer: Option B
Explanation:
 
 
Given Expression =
(a2
- b2
)
=
(a2
- b2
)
= 1.
(a + b)(a - b) (a2
- b2
)
31. If
144
=
14.4
, then the value of x is:
0.144 x
A. 0.0144 B. 1.44
C. 14.4 D. 144
Answer: Option A
Explanation:
144
=
14.4
0.144 x
144 x 1000
=
14.4
144 x
x =
14.4
= 0.0144
1000
LOGICAL REASONING:
Number Series
32. Look at this series: 2, 1, (1/2), (1/4), ... What number should come next?
A. (1/3) B. (1/8)
C. (2/8) D. (1/16)
 
 
Answer: Option B
Explanation:
This is a simple division series; each number is one-half of the previous number.
In other terms to say, the number is divided by 2 successively to get the next result.
4/2 = 2
2/2 = 1
1/2 = 1/2
(1/2)/2 = 1/4
(1/4)/2 = 1/8 and so on.
33. Look at this series: 58, 52, 46, 40, 34, ... What number should come next?
A. 26 B. 28
C. 30 D. 32
Answer: Option B
Explanation:
This is a simple subtraction series. Each number is 6 less than the previous number.
34. Look at this series: 3, 4, 7, 8, 11, 12, ... What number should come next?
A. 7 B. 10
C. 14 D. 15
Answer: Option D
Explanation:
This alternating addition series begins with 3; then 1 is added to give 4; then 3 is added to give 7;
then 1 is added, and so on.
 
 
35. Look at this series: 31, 29, 24, 22, 17, ... What number should come next?
A. 15 B. 14
C. 13 D. 12
Answer: Option A
Explanation:
This is a simple alternating subtraction series, which subtracts 2, then 5.
DATA INTERPRETATION
36. Bar Charts
1. What is the average sales of all the branches (in thousand numbers) for the year 2000?
A. 73 B. 80
C. 83 D. 88
 
 
Answer: Option B
Explanation:
Average sales of all the six branches (in thousand numbers) for the year 2000
=
1
x [80 + 75 + 95 + 85 + 75 + 70]
6
= 80.
2. Total sales of branches B1, B3 and B5 together for both the years (in thousand numbers) is?
A. 250 B. 310
C. 435 D. 560
Answer: Option D
Explanation:
Total sales of branches B1, B3 and B5 for both the years (in thousand numbers)
= (80 + 105) + (95 + 110) + (75 + 95)
= 560.
3. What is the ratio of the total sales of branch B2 for both years to the total sales of branch B4
for both years?
A. 2:3 B. 3:5
C. 4:5 D. 7:9
Answer: Option D
Explanation:
 
 
Required ratio =
(75 + 65)
=
140
=
7
.
(85 + 95) 180 9
NON VERBAL REASONING
Series
37. Select a figure from amongst the Answer Figures which will continue the same series as
established by the five Problem Figures.
Problem Figures: Answer Figures:
(A) (B) (C) (D) (E) (1) (2) (3) (4) (5)
A. 1 B. 2
C. 3 D. 4
E. 5
Answer: Option A
Explanation:
In each step, the dot moves one space CW and the arrow moves two spaces CW.
38. Select a figure from amongst the Answer Figures which will continue the same series as
established by the five Problem Figures.
Problem Figures: Answer Figures:
(A) (B) (C) (D) (E) (1) (2) (3) (4) (5)
A. 1 B. 2
C. 3 D. 4
E. 5
 
 
Answer: Option C
Explanation:
The pin rotates 45o
CW and 90o
CW alternately and moves one space (each space is equal to half-a-
side of the square) and two spaces CW alternately. The arrow rotates 90o
ACW and 45o
ACW
alternately and moves two spaces and one space.
39. Select a figure from amongst the Answer Figures which will continue the same series as
established by the five Problem Figures.
Problem Figures: Answer Figures:
(A) (B) (C) (D) (E) (1) (2) (3) (4) (5)
A. 1 B. 2
C. 3 D. 4
E. 5
Answer: Option C
Explanation:
In each step, the pin rotates 90o
CW and the arrow rotates 90o
ACW.
40. Select a figure from amongst the Answer Figures which will continue the same series as
established by the five Problem Figures.
Problem Figures: Answer Figures:
(A) (B) (C) (D) (E) (1) (2) (3) (4) (5)
A. 1 B. 2
 
 
C. 3 D. 4
E. 5
Answer: Option D
Explanation:
In each step, all the elements move to the adjacent corner (of the square boundary) in a CW
direction and the element that reaches the upper-left corner gets vertically inverted.
42. Select a figure from amongst the Answer Figures which will continue the same series as
established by the five Problem Figures.
Problem Figures: Answer Figures:
(A) (B) (C) (D) (E) (1) (2) (3) (4) (5)
A. 1 B. 2
C. 3 D. 4
E. 5
Answer: Option D
Explanation:
In each step, the first element moves to the third position and gets replaced by a new element; the
second and the third elements move to the first and the second positions respectively and the
entire figure rotates 90o
CW.
Analytical Reasoning
41. Find the number of triangles in the given figure.
A. 8 B. 10
 
 
C. 12 D. 14
Answer: Option D
Explanation:
42. The figure may be labelled as shown.
The simplest triangles are AHG, AIG, AIB, JFE, CJE and CED i.e. 6 in number.
The triangles composed of two components each are ABG, CFE, ACJ and EGI i.e. 4 in number.
The triangles composed of three components each are ACE, AGE and CFD i.e. 3 in number.
There is only one triangle i.e. AHE composed of four components.
Therefore, There are 6 + 4 + 3 + 1 = 14 triangles in the given figure.
42. Find the minimum number of straight lines required to make the given figure.
A. 13 B. 15
C. 17 D. 19
Answer: Option A
Explanation:
 
 
The figure may be labelled as shown.
The horizontal lines are IJ, AB, EF, MN, HG, DC and LK i.e. 7 in number.
The vertical lines are AD, EH, IL, FG, BC and JK i.e. 6 in number.
Thus, there are 7 + 6 = 13 straight lines in the figure.
43. Find the number of triangles in the given figure.
A. 4 B. 5
C. 6 D. 7
Answer: Option B
Explanation:
The figure may be labelled as shown.
The simplest triangles are ADE, BDF, DEF and EFC i.e. 4 in number.
 
 
There is only one triangle ABC composed of four components.
Thus, there are 4+1 = 5 triangles in the given figure.
Mirror Images
VERBAL REASONING
Series Completion
44. 120, 99, 80, 63, 48, ?
A. 35 B. 38
C. 39 D. 40
Answer: Option A
Explanation:
The pattern is - 21, - 19, - 17, - 15,.....
So, missing term = 48 - 13 = 35.
45. 3, 10, 101,?
A. 10101 B. 10201
C. 10202 D. 11012
Answer: Option C
Explanation:
Each term in the series is obtained by adding 1 to the square of the preceding term.
So, missing term = (101)2
+ 1 = 10202.
46. 125,80,45,20,?
 
 
A. 5 B. 8
C. 10 D. 12
Answer: Option A
Explanation:
The pattern is - 45, - 35, - 25, .....
So, missing term = 20 - 15 = 5.
Seating Arrangement
47. A, P, R, X, S and Z are sitting in a row. S and Z are in the centre. A and P are at the ends. R is
sitting to the left of A. Who is to the right of P ?
A. A B. X
C. S D. Z
Answer: Option B
Explanation:
The seating arrangement is as follows:
Therefore, right of P is X.
48. A, B, C, D and E are sitting on a bench. A is sitting next to B, C is sitting next to D, D is not sitting
with E who is on the left end of the bench. C is on the second position from the right. A is to the
right of B and E. A and C are sitting together. In which position A is sitting ?
A. Between B and D B. Between B and C
C. Between E and D D. Between C and E
 
 
Answer: Option B
Explanation:
Therefore, A is sitting in between B and C.
49. Which one is immediate right to the V ?
A. P B. U
C. R D. T
Answer: Option D
Explanation:
T is immediate right to the V.
50. Who is sitting immediate right to Reeta ?
A. Bindu B. Rani
C. Mary D. Seema
Answer: Option C
Explanation:
 
 
Mary is sitting immediate right to Reeta.
DATA INT
51. Study the following table and answer the questions based on it.
Expenditures of a Company (in Lakh Rupees) per Annum Over the given Years.
Year
Item of Expenditure
Salary Fuel and Transport Bonus Interest on Loans Taxes
1998 288 98 3.00 23.4 83
1999 342 112 2.52 32.5 108
2000 324 101 3.84 41.6 74
2001 336 133 3.68 36.4 88
2002 420 142 3.96 49.4 98
1. What is the average amount of interest per year which the company had to pay during this
period?
A. Rs. 32.43 lakhs B. Rs. 33.72 lakhs
C. Rs. 34.18 lakhs D. Rs. 36.66 lakhs
2. What is the average amount of interest per year which the company had to pay during this
period?
A. Rs. 32.43 lakhs B. Rs. 33.72 lakhs
C. Rs. 34.18 lakhs D. Rs. 36.66 lakhs
Answer: Option D
Explanation:
 
 
Average amount of interest paid by the Company during the given period
= Rs.
23.4 + 32.5 + 41.6 + 36.4 + 49.4
lakhs
5
= Rs.
183.3
lakhs
5
= Rs. 36.66 lakhs.
2. The total amount of bonus paid by the company during the given period is approximately what
percent of the total amount of salary paid during this period?
A. 0.1% B. 0.5%
C. 1% D. 1.25%
Answer: Option C
Explanation:
Required percentage =
(3.00 + 2.52 + 3.84 + 3.68 + 3.96)
x 100 %
(288 + 342 + 324 + 336 + 420)
=
17
x 100 %
1710
1%.
3. Total expenditure on all these items in 1998 was approximately what percent of the total
expenditure in 2002?
A. 62% B. 66%
C. 69% D. 71%
Answer: Option C
Explanation:
Required percentage = (288 + 98 + 3.00 + 23.4 + 83) x 100 %
 
 
(420 + 142 + 3.96 + 49.4 + 98)
=
495.4
x 100 %
713.36
69.45%.
4. The total expenditure of the company over these items during the year 2000 is?
A. Rs. 544.44 lakhs B. Rs. 501.11 lakhs
C. Rs. 446.46 lakhs D. Rs. 478.87 lakhs
Answer: Option A
Explanation:
Total expenditure of the Company during 2000
= Rs. (324 + 101 + 3.84 + 41.6 + 74) lakhs
= Rs. 544.44 lakhs.
5. The ratio between the total expenditure on Taxes for all the years and the total expenditure on
Fuel and Transport for all the years respectively is approximately?
A. 4:7 B. 10:13
C. 15:18 D. 5:8
Answer: Option B
Explanation:
Required ratio =
(83 + 108 + 74 + 88 + 98)
(98 + 112 + 101 + 133 + 142)
=
451
586
 
 
=
1
1.3
=
10
.
13
52. Study the following table and answer the questions.
Number of Candidates Appeared and Qualified in a Competitive Examination from Different States
Over the Years.
State
Year
1997 1998 1999 2000 2001
App. Qual. App. Qual. App. Qual. App. Qual. App. Qual.
M 5200 720 8500 980 7400 850 6800 775 9500 1125
N 7500 840 9200 1050 8450 920 9200 980 8800 1020
P 6400 780 8800 1020 7800 890 8750 1010 9750 1250
Q 8100 950 9500 1240 8700 980 9700 1200 8950 995
R 7800 870 7600 940 9800 1350 7600 945 7990 885
Total number of candidates qualified from all the states together in 1997 is approximately what
percentage of the total number of candidates qualified from all the states together in 1998?
A. 72% B. 77%
C. 80% D. 83%
Answer: Option C
Explanation:
Required percentage =
(720 + 840 + 780 + 950 + 870)
x 100 %
(980 + 1050 + 1020 + 1240 + 940)
 
 
=
4160
x 100 %
5230
= 79.54% 80%.
What is the average candidates who appeared from State Q during the given years?
A. 8700 B. 8760
C. 8990 D. 8920
Answer: Option C
Explanation:
Required average =
8100 + 9500 + 8700 + 9700 + 8950
5
=
44950
5
= 8990.
In which of the given years the number of candidates appeared from State P has maximum percentage
of qualified candidates?
A. 1997 B. 1998
C. 1999 D. 2001
Answer: Option D
Explanation:
The percentages of candidates qualified to candidates appeared from State P during different years
are:
For 1997
780
x 100 % = 12.19%.
6400
For 1998
1020
x 100 % = 11.59%.
8800
For 1999
890
x 100 % = 11.41%.
7800
 
 
For 2000
1010
x 100 % = 11.54%.
8750
For 2001
1250
x 100 % = 12.82%.
9750
Maximum percentage is for the year 2001.
What is the percentage of candidates qualified from State N for all the years together, over the
candidates appeared from State N during all the years together?
A. 12.36% B. 12.16%
C. 11.47% D. 11.15%
Answer: Option D
Explanation:
Required percentage =
(840 + 1050 + 920 + 980 + 1020)
x 100 %
(7500 + 9200 + 8450 + 9200 + 8800)
=
4810
x 100 %
43150
= 11.15%
The percentage of total number of qualified candidates to the total number of appeared candidates
among all the five states in 1999 is?
A. 11.49% B. 11.84%
C. 12.21% D. 12.57%
Answer: Option B
Explanation:
Required percentage =
(850 + 920 + 890 + 980 + 1350)
x 100 %
(7400 + 8450 + 7800 + 8700 + 9800)
=
4990
x 100 %
42150
 
 
= 11.84%.
53. The following table gives the percentage of marks obtained by seven students in six different
subjects in an examination.
The Numbers in the Brackets give the Maximum Marks in Each Subject.
Student
Subject (Max. Marks)
Maths Chemistry Physics Geography History
Computer
Science
(150) (130) (120) (100) (60) (40)
Ayush 90 50 90 60 70 80
Aman 100 80 80 40 80 70
Sajal 90 60 70 70 90 70
Rohit 80 65 80 80 60 60
Muskan 80 65 85 95 50 90
Tanvi 70 75 65 85 40 60
Tarun 65 35 50 77 80 80
What are the average marks obtained by all the seven students in Physics? (rounded off to two digit
after decimal)
A. 77.26 B. 89.14
C. 91.37 D. 96.11
Answer: Option B
Explanation:
Average marks obtained in Physics by all the seven students
=
1
x [ (90% of 120) + (80% of 120) + (70% of 120)
7
+ (80% of 120) + (85% of 120) + (65% of 120) + (50% of 120) ]
= 1 x [ (90 + 80 + 70 + 80 + 85 + 65 + 50)% of 120 ]
 
 
7
=
1
x [ 520% of 120 ]
7
=
624
7
= 89.14.
The number of students who obtained 60% and above marks in all subjects is?
A. 1 B. 2
C. 3 D. None
Answer: Option B
Explanation:
From the table it is clear that Sajal and Rohit have 60% or more marks in each of the six subjects.
What was the aggregate of marks obtained by Sajal in all the six subjects?
A. 409 B. 419
C. 429 D. 449
Answer: Option D
Explanation:
Aggregate marks obtained by Sajal
= [ (90% of 150) + (60% of 130) + (70% of 120)
+ (70% of 100) + (90% of 60) + (70% of 40) ]
= [ 135 + 78 + 84 + 70 + 54 + 28 ]
= 449.
In which subject is the overall percentage the best?
 
 
A. Maths B. Chemistry
C. Physics D. History
Answer: Option A
Explanation:
We shall find the overall percentage (for all the seven students) with respect to each subject.
The overall percentage for any subject is equal to the average of percentages obtained by all the
seven students since the maximum marks for any subject is the same for all the students.
Therefore, overall percentage for:
(i) Maths =
1
x (90 + 100 + 90 + 80 + 80 + 70 + 65) %
7
=
1
x (575) %
7
= 82.14%.
(ii) Chemistry =
1
x (50 + 80 + 60 + 65 + 65 + 75 + 35) %
7
=
1
x (430) %
7
= 61.43%.
(iii) Physics =
1
x (90 + 80 + 70 + 80 + 85 + 65 + 50) %
7
=
1
x (520) %
7
= 74.29%.
(iv) Geography =
1
x (60 + 40 + 70 + 80 + 95 + 85 + 77) %
7
 
 
=
1
x (507) %
7
= 72.43%.
(v) History =
1
x (70 + 80 + 90 + 60 + 50 + 40 + 80) %
7
=
1
x (470) %
7
= 67.14%.
(vi) Comp. Science =
1
x (80 + 70 + 70 + 60 + 90 + 60 + 80) %
7
=
1
x (510) %
7
= 72.86%.
Clearly, this percentage is highest for Maths.
What is the overall percentage of Tarun?
A. 52.5% B. 55%
C. 60% D. 63%
Answer: Option C
Explanation:
Aggregate marks obtained by Tarun
= [ (65% of 150) + (35% of 130) + (50% of 120)
+ ((77% of 100) + (80% of 60) + (80% of 40) ]
= [ 97.5 + 45.5 + 60 + 77 + 48 + 32 ]
= 360.
 
 
The maximum marks (of all the six subjects)
= (150 + 130 + 120 + 100 + 60 + 40)
= 600.
Overall percentage of Tarun =
360
x 100 % = 60%.
600
PERCENTAGE:
54. A student multiplied a number by
3
instead of
5
.
5 3
What is the percentage error in the calculation?
A. 34% B. 44%
C. 54% D. 64%
Answer: Option D
Explanation:
Let the number be x.
Then, error =
5
x -
3
x =
16
x.
3 5 15
Error% =
16x
x
3
x 100
% = 64%.15 5x
55. Two tailors X and Y are paid a total of Rs. 550 per week by their employer. If X is paid 120 percent of
the sum paid to Y, how much is Y paid per week?
A. Rs. 200 B. Rs. 250
C. Rs. 300 D. None of these
Answer: Option B
 
 
Explanation:
Let the sum paid to Y per week be Rs. z.
Then, z + 120% of z = 550.
z +
120
z = 550
100
11
z = 550
5
z =
550 x 5
= 250.11
56. Gauri went to the stationers and bought things worth Rs. 25, out of which 30 paise went on sales tax
on taxable purchases. If the tax rate was 6%, then what was the cost of the tax free items?
A. Rs. 15 B. Rs. 15.70
C. Rs. 19.70 D. Rs. 20
Answer: Option C
Explanation:
Let the amount taxable purchases be Rs. x.
Then, 6% of x =
30
100
x =
30
x
100
= 5.100 6
Cost of tax free items = Rs. [25 - (5 + 0.30)] = Rs. 19.70
57. Rajeev buys good worth Rs. 6650. He gets a rebate of 6% on it. After getting the rebate, he pays
sales tax @ 10%. Find the amount he will have to pay for the goods.
A. Rs. 6876.10 B. Rs. 6999.20
C. Rs. 6654 D. Rs. 7000
Answer: Option A
Explanation:
 
 
Rebate = 6% of Rs. 6650 = Rs.
6
x 6650 = Rs. 399.
100
Sales tax = 10% of Rs. (6650 - 399) = Rs.
10
x 6251 = Rs. 625.10
100
Final amount = Rs. (6251 + 625.10) = Rs. 6876.10
----------------------
58. The ages of the two persons differ by 20 years. If 5 year ago, the older one be 5 times as old
as the younger one, then their present ages, in year are:
A. 25, 5,
B. 30, 10
C. 35, 15
D. 50, 30
59. In what ratio must rice at Rs 9.30 per Kg be mixed with rice at Rs 10.80 per Kg so that the
mixture be worth Rs 10 per Kg?
A. 6:5
B. 8:7
C. 3:7
D. 6:1
Answer – (B)
Solution:
C.P of 1 Kg C.P of 1 Kg
rice of 1st rice of 2nd
kind (930p) kind (1080p)
 /
Mean Price
(1000p)
/ 
(1080 – 1000) : (1000 - 930)
80 70
Thus, required ratio = 80 : 70 = 8 : 7
 
 
60. How much water must be added to 60 litres of milk at 1.5 litres for Rs. 20 So as to have a
mixture worth Rs.10 2/3 a litre?
A. 10 litres
B. 12 litres
C. 15 litres
✖ D. 18 litres
Answer – (C)
Solution:
C.P. of 1 litre of milk =Rs.
(20x2)
3
=Rs.
40
3
C.P. of 1 litre of water = 0
Mean price = Rs.
32
3
By the rule of alligation, we have:
C.P of 1 litre C.P of 1 litre
Of water of milk
(0) (Rs. 40/3)
 /
Mean Price
(Rs. 32/3)
/ 
(
40
3
–
 
 
32
3
) (
32
3−0
)
8/3 32/3
Ratio of water and milk =
8
3
:
32
3
=8:32=1:4
Thus, Quantity of water to be added to 60 litres of milk = (
1
4
×60)litres= 15 litres
61. In what ratio must wheat at Rs.3.20 per kg be mixed with wheat at Rs.2.90 per kg so that the
mixture be worth Rs.3.08 per kg?
A. 3 : 4
B. 2 : 3
C. 3 : 2
✖ D. 4 : 3
Answer – (C)
Solution:
C.P of a unit quantity of 1st kind = Rs. 3.20
C.P of a unit quantity of 2nd kind = Rs. 2.90
Mean price = Rs.3.08
C.P of unit quantity C.P of unit quantity
of 1st kind of 2nd kind
(Rs. 3.20) (Rs. 2.90)
 
 
 /
Mean Price
(Rs.3.08)
/ 
(3.08 – 2.90) : (3.20 – 3.08)
0.18 0.12
Required ratio = 0.18 : 0.12 = 3 : 2
62. In what proportion must rice at Rs. 3.10 per kg be mixed with rice at Rs. 3.60 per kg so that
the mixture be worth Rs. 3.25 per kg?
A. 3 : 7
B. 5 : 3
C. 3 : 5
✔ D. 7 : 3
Answer – (D)
Solution:
C.P of a unit quantity of 1st kind = 310p (in paise)
C.P of a unit quantity of 2nd kind = 360p
Mean price = 325p
C.P of unit quantity C.P of unit quantity
of 1st kind of 2nd kind
(310p) (360p)
 /
Mean Price
(325p)
/ 
(360 – 325) : (325 – 310)
35 15
Required ratio = 35 : 15 = 7 : 3
They must be mixed in the ratio 7 : 3
AVERAGE:
 
 
68. The average of first five multiples of 3 is :
A. 9
B. 10
C. 8
✖ D. 11
Answer – (A)
Solution:
Basic Formula:
1, 2, 3 ..… n
If n is odd, the formula is (
n+1
2
)th term
The five multiples of 3 is 3, 6, 9, 12, 15
(
n+1
2
) ⇒ (
5+1
2
)th term
⇒
6
2
th term =3rd term
Here 3rd term is 9
69. There are two sections A and B of a class, consisting of 36 and 44 students’ respectively. If
the average weight of section A is 40kg and that of section B is 35kg, find the average weight of
the whole class.
A. 30 kg
B. 35 kg
C. 42.5 kg
D. 37.25 kg
 
 
Answer – (D)
Solution:
Total weight of (36+44) Students = (36x40+44x35)kg = 2980kg
Therefore average weight of the whole class = (2980/80) kg
Therefore average weight = 37.25kg
Distance between two stations A and B is 778km. A train covers the journey from A to B at
84km per hour and returns back to A with a uniform speed of 56km per hour. Find the average
speed of train during the whole journey.
A. 60 km/hr
B. 30.5 km/hr
C. 57 km/hr
✔ D. 67.2 km/hr
Answer - (D)
Solution:
Average speed=(
2xy
x+y
)km/hr
=(
2×84×56
84+56
) km/hr
=(
2×84×56
140
)km/hr
= 67.2km/hr.
APTITUDE:
Exam2win > Aptitude Question Papers (All Topics)
 
 
APTITUDE QUESTIONS AND ANSWERS 1 to 10
70. 1. A man lends Rs. 10000 in four parts. If be gets 8% on Rs. 2000 7 on Rs. 4000 and 8 on Rs. 1400
what per cent must he get for the remainder, if the average interest is 8.13%?
7%
9%
9
10
ANSWER : 9%
71. On july 2, 1985 it was Wednesday. The day of the week on July 2, 1984 was:
Wednesday
Tuesday
Monday
Thursday
ANSWER : Monday
72. A tank can be filled by one tap in 20 min, and by another in 25 min. both the taps are kept open
for 5 min, and then the second is turned off. In how many minutes more is the tank completely filled?
17 min
12 min
11 min
6 min
ANSWER : 1 min
 
 
73. If each side of a cube is doubled, then its volume :
is doubled
becomes 4 times
becomes 6 times
becomes 8 times
ANSWER : becomes 8 times
74. A sum of money is sufficient to pay A’s wages for 21 days or B’s wages for 28 days. The money
is sufficient to pay the wages of both for:
12 days
12 days
14 days
none of these
ANSWER : 12 days
75. The mean proportional between 0.32 and 0.02 is:
0.34
0.3
0.16
0.08
ANSWER : .08
76. The ratio of milk and water in 66 kg. of adulterated milk is 5 : 1. Water is added to it to make the
ratio 5 : 3. The quantity of water added is :
22 kg.
24.750 kg.
 
 
16.500 kg.
20 kg.
ANSWER : 22 kg.
77. The length of hall is 4/3) times its breadth. If the area of the hall be 300 square metres the
difference between the length and the breadth is:
15 metres
4 metres
3 metres
none of these
ANSWER : none of these
78. A can run a kilometer in 4 min. 54 sec. and B in 5 min. How many metres start can A give B in a
km race so that the race may end in a dead heat?
20 metres
16 metres
18 metres
14.5 metres
ANSWER : 20 metres
79. A,B,C, enter into a partnership and their capitals are in the proportion of . A withdraws half his
capital at the end of 4 months. Out of a total annual profit of Rs. 847, A’ share is:
Rs. 252
Rs. 280
Rs.315
Rs. 412
ANSWER : Rs. 280
 
 
AVERAGE
APTITUDE QUESTIONS AND ANSWERS 21 to 30
80. The average age of an adult class is 40 years. 12 new students with an average age of 32 years
join the class, thereby decreasing the average by 4 years. The original strength of the class was :
10
11
12
15
ANSWER : 12
81. A cistern is normally filled in 8 hours but takes two hours longer to fill because of a leak in it
sbottom. If the cistern is full, the leak will empty it in:
16 hrs
20 hrs
25 hrs
40 hrs
ANSWER : 0 hrs
82. The difference between simple interest and compound interest on a sum for 2 years at 8% when
the interest is compounded annually is Rs.16. If the interest were compounded half yearly, the
difference in two interests would be nearly:
Rs.16
Rs.16.80
Rs. 21.85
RS.24.64
ANSWER : RS.24.64
83. At what time between 9 and 10 will the hands of a watch be together ?
45 minutes past 9
50 minutes past 9
49 minutes past 9
 
 
48 minutes past 9
ANSWER : 49 minutes past 9
84. If I purchased 11 books for Rs. 10 and sold all the books at the rate of 10 books for Rs. 11, the
profit per cent is:
10%
11%
21%
100%
ANSWER : 21%
85. A fraction becomes 4 when 1 is added to the both the numerator and denominator; and it
becomes 7 when 1 is subtracted from both the numerator and denominator. The numerator of the
given fraction is:
2
3
7
15
ANSWER : 15
85. The current of a stream runs at 1 km/hr. A motor boat goes 35 km upstream and back again to
the starting point in 12 hours. The speed of motorboat in still water is :
6 km/hr
7 km/hr
8.5 km/hr
8 km/hr
ANSWER : 6 km/hr
86. A man sold 20 articles for Rs. 60 and gained 20%. How many articles did he buy for Rs.60?
22
24
25
26
ANSWER : 24
 
 
87. X an Y are two stations 500 km apart. A train starts form X and moves towards Y at 20 km/hr.
Another rain starts from Y at the same time and moves towards X at 30 km/hr. How far from X will
they cross each other:
200 km
300 km
120 km
40 km
ANSWER : 200 km
88. P.V. Narsimha Rao was elected party leader on 29 th may. 1991. What was the day of the week?
Tuesday
Friday
Wednesday
Sunday
ANSWER : Wednesday
AVERAGE
APTITUDE QUESTIONS AND ANSWERS 21 to 30
89. The average age of an adult class is 40 years. 12 new students with an average age of 32 years
join the class, thereby decreasing the average by 4 years. The original strength of the class was :
10
11
12
15
ANSWER : 12
90. A cistern is normally filled in 8 hours but takes two hours longer to fill because of a leak in it
sbottom. If the cistern is full, the leak will empty it in:
16 hrs
20 hrs
25 hrs
 
 
40 hrs
ANSWER : 0 hrs
91. The difference between simple interest and compound interest on a sum for 2 years at 8% when
the interest is compounded annually is Rs.16. If the interest were compounded half yearly, the
difference in two interests would be nearly:
Rs.16
Rs.16.80
Rs. 21.85
RS.24.64
ANSWER : RS.24.64
92. At what time between 9 and 10 will the hands of a watch be together ?
45 minutes past 9
50 minutes past 9
49 minutes past 9
48 minutes past 9
ANSWER : 49 minutes past 9
93. If I purchased 11 books for Rs. 10 and sold all the books at the rate of 10 books for Rs. 11, the
profit per cent is:
10%
11%
21%
100%
ANSWER : 21%
94. A fraction becomes 4 when 1 is added to the both the numerator and denominator; and it
becomes 7 when 1 is subtracted from both the numerator and denominator. The numerator of the
given fraction is:
2
3
7
15
 
 
ANSWER : 15
95. The current of a stream runs at 1 km/hr. A motor boat goes 35 km upstream and back again to
the starting point in 12 hours. The speed of motorboat in still water is :
6 km/hr
7 km/hr
8.5 km/hr
8 km/hr
ANSWER : 6 km/hr
96. A man sold 20 articles for Rs. 60 and gained 20%. How many articles did he buy for Rs.60?
22
24
25
26
ANSWER : 24
97. X an Y are two stations 500 km apart. A train starts form X and moves towards Y at 20 km/hr.
Another rain starts from Y at the same time and moves towards X at 30 km/hr. How far from X will
they cross each other:
200 km
300 km
120 km
40 km
ANSWER : 200 km
98. P.V. Narsimha Rao was elected party leader on 29 th may. 1991. What was the day of the week?
Tuesday
Friday
Wednesday
Sunday
ANSWER : Wednesday
 
 
VOLUME AND SURFACE AREA QUESTIONS AND ANSWERS 1 to 3
99. The length of diagonal of a cube is (14 x ) cm. The volume of the cube is :
2744 cm3
2744 cm3
588 cm3
3528 cm3
ANSWER : 2744 cm3
100. The surface area of a cuboid 22 cm by 12 cm by 7.5 cm, is :
1980 cm2
2076 cm2
1038 cm2
none of these
ANSWER : 1038 cm2
101. Given that 1 cubic cm of marble weight 25 gms, the weight of a marble block 28 cm in width
and 5 cm thick is 112 kg. the length of the block is :
36 cm
37.5 cm
32 cm
26.5 cm
ANSWER : 32 cm
PROBLEMS ON NUMBERS QUESTIONS AND ANSWERS 1 to 3
102. If one-fourth of one-third of one-half of a number is 15, the number is:
72
120
180
360
ANSWER : 360
 
 
103. Three numbers are in the ratio 3:4:5. The sum of the largest and the smallest equals the sum of
the third and 52. The smallest number is:
20
27
39
52
ANSWER : 39
104. The sum of three consecutive odd numbers is 57. The middle one is:
19
21
23
17
ANSWER : 19
PARTNERSHIP QUESTIONS AND ANSWERS 1 to 3
105. Jayant started a business, investing Rs. 6000. Six months later madhu joined him, investing Rs.
4000, If they made a profit of Rs. 5200 at the end of the year, how much must be the share of Madhu?
Rs. 2080
Rs.1300
Rs.1800
Rs. 2600
ANSWER : Rs.1800
106. A and B invest in business in the ratio 3 :2. If the 5% of the total profit goes to charity and A?s
share is Rs. 855, total profit is:
Rs.1576
Rs. 1537.50
Rs.1500
Rs.1425
ANSWER : Rs. 1500
 
 
107. A, B and c invest Rs. 2000, Rs.3000 and Rs. 4000 in a business, After one year, A removed his
money but B and C continued for one more year. If the net profit after 2 years be Rs.3200, them A?s
share in the Profit is:
Rs. 1000
Rs. 600
Rs. 800
Rs. 400
ANSWER : Rs. 400
108. Which of the following diagrams indicates the best relation between Travelers, Train and Bus ?
A. B.
C. D.
Answer & Explanation
Answer: Option C
Explanation:
Bus and Train are different from each other but some travelers travel by bus and some travel by
train.
109. Which of the following diagrams indicates the best relation between Profit, Dividend and
Bonus ?
A. B.
C. D.
Answer & Explanation
Answer: Option B
Explanation:
Bonus and Dividend are different from each other. But both these are parts of profit
 
 
110. Which of the following diagrams indicates the best relation between Women, Mothers and
Engineers ?
A. B.
C. D.
Answer & Explanation
Answer: Option A
Explanation:
All mothers are women and some mothers and some women may be engineers.
111. Which of the following diagrams indicates the best relation between Factory, Product and
Machinery ?
A. B.
C. D.
Answer & Explanation
Answer: Option D
Explanation:
Product and Machinery are different from each other but both are found in Factory.
112. Which of the following diagrams indicates the best relation between Author, Lawyer and
Singer ?
A. B.
C. D.
Answer & Explanation
Answer: Option B
 
 
Explanation:
All the three are different professions.
113. Which of the following diagrams indicates the best relation between Judge, Thieves and
Criminals ?
A. B.
C. D.
Answer & Explanation
Answer: Option B
Explanation:
All the thieves are criminals while judge is different from these.
114. Which of the following diagrams indicates the best relation between Judge, Thieves and
Criminals ?
A. B.
C. D.
116. Which of the following diagrams indicates the best relation between Pigeon, Bird and Dog ?
A. B.
C. D.
Answer & Explanation
Answer: Option A
Explanation:
All the pigeons are birds while dog is different from these.
 
 
117. Which of the following diagrams indicates the best relation between Earth, Sea and Sun ?
A. B.
C. D.
Answer & Explanation
Answer: Option A
Explanation:
Sea is a part of Earth while Sun is different from these two.
118. Which of the following diagrams indicates the best relation between Hockey, Football and
Cricket ?
A. B.
C. D.
Answer & Explanation
Answer: Option B
Explanation:
All these three games are different from each other.
119. One morning Udai and Vishal were talking to each other face to face at a crossing. If Vishal's
shadow was exactly to the left of Udai, which direction was Udai facing?
A. East B. West
C. North D. South
Answer & Explanation
Answer: Option C
Explanation:
 
 
120. Y is in the East of X which is in the North of Z. If P is in the South of Z, then in which direction
of Y, is P?
A. North B. South
C. South-East D. None of these
Answer & Explanation
Answer: Option D
Explanation:
P is in South-West of Y.
121. If South-East becomes North, North-East becomes West and so on. What will West become?
A. North-East B. North-West
C. South-East D. South-West
Answer & Explanation
Answer: Option C
Explanation:
It is clear from the diagrams that new name of West will become South-East.
 
 
121. A man walks 5 km toward south and then turns to the right. After walking 3 km he turns to
the left and walks 5 km. Now in which direction is he from the starting place?
A. West B. South
C. North-East D. South-West
Answer: Option D
Explanation:
Hence required direction is South-West.
123. Rahul put his timepiece on the table in such a way that at 6 P.M. hour hand points to North. In
which direction the minute hand will point at 9.15 P.M. ?
A. South-East B. South
C. North D. West
Answer & Explanation
Answer: Option D
Explanation:
At 9.15 P.M., the minute hand will point towards west.
 
 
124. Dev, Kumar, Nilesh, Ankur and Pintu are standing facing to the North in a playground such as
given below:
1. Kumar is at 40 m to the right of Ankur.
2. Dev is are 60 m in the south of Kumar.
3. Nilesh is at a distance of 25 m in the west of Ankur.
4. Pintu is at a distance of 90 m in the North of Dev.
1. Which one is in the North-East of the person who is to the left of Kumar?
A. Dev B. Nilesh
C. Ankur D. Pintu
Answer & Explanation
Answer: Option D
Explanation:
Ankur is in the left of Kumar. Hence Pintu is in North-East of Ankur.
124. Each of the following questions is based on the following information:
1. Six flats on a floor in two rows facing North and South are allotted to P, Q, R, S, T and U.
2. Q gets a North facing flat and is not next to S.
3. S and U get diagonally opposite flats.
4. R next to U, gets a south facing flat and T gets North facing flat.
1. If the flats of P and T are interchanged then whose flat will be next to that of U?
A. P B. Q
C. R D. T
Answer & Explanation
 
 
Answer: Option C
Explanation:
Hence flat R will be next to U.
126. Rasik walked 20 m towards north. Then he turned right and walks 30 m. Then he turns right
and walks 35 m. Then he turns left and walks 15 m. Finally he turns left and walks 15 m. In which
direction and how many metres is he from the starting position?
A. 15 m West B. 30 m East
C. 30 m West D. 45 m East
Answer & Explanation
Answer: Option D
Explanation:
 
 
127. Solution:
Hank: pear apple cherry rose
Sam: cherry onion rose tulip
Paul: carrot gourd onion rose
Zick: aster rose tulip lily
Luke: pear nut gourd parsley
128. Solution:
The Spanish ship goes to Port Said and the French ship carries tea. However, tea can be carried by
the Brazilian ship, too.
If you understood position 'to the right' to mean anywhere on the right side from the given point
(not only right next to).
French 5:00 tea blue Genoa
Greek 6:00 coffee red Hamburg
Brazilian 8:00 cocoa black Manila
English 9:00 rice white Marseille
Spanish 7:00 corn green Port Said
 
 
129. Solution:
1 2 3 4 5
Name Bob Rachael Eilish Keeley Amy
Favorite TV
programme
The Simpsons
Coronation
Street
Desperate
Housewives
Neighbours Eastenders
Destination France Italy Africa England Australia
Age 46 21 81 52 14
Where they live Town Youth Hostel Farm Village City
Hairstyle Bald Long Afro Curly Straight
130. Solution:
Holly did!
See below for full solution:
Person Alex Jackie Julieanne Cameron Holly
Flavor Vanilla Strawberry Banana Caramel Chocolate
Cookie Jar Mini Brass Tall Square Round
Brand Coles Dick Smith Paradise No Frills Arnotts
Cookie Choc chip Tiny Teddies 100s and 1000s Oreos Ginger
Place Lounge Bedroom Kitchen Closet Table
131. Solution:
Betty, Carol and Dan work in Administration department.
Earl works in the Marketing department.
132.
house 1 2 3 4 5
color yellow blue red green white
nationality norweigian dane brit german swede
drink water tea milk coffee beer
 
 
smoke dunhill blend pall mall prince bluemaster
pet cats horse birds fish dogs
134. Solution
 888 + 88 + 8 + 8 + 8
 (8(8(8+8)-(8+8)/8))-8
 8888/8.888
 (888-8) + 8×(8+8) – 8
 ((8×(8+8))-((8+8+8)/8))*8
 (8+((8+8)/8))^((8+8+8)/8)
 (8+((8+8)/8))^((88/8)-8)
 ((8×(8+8))-((88/8)-8))×8
 (8888-888)/8
 8(8×8+8×8)-8-8-8
135. Solution:
Tom's Granny is 58 years old. Let's see why:
First child born: Granny is 19
Second child born: Granny is 23 (19 + 4)
Third child born: Granny is 27 (23 + 4)
Fourth child born: Granny is 31 (27 + 4)
Fifth child born: Granny is 35 (31 + 4)
Sixth child born: Granny is 39 (35 + 4)
Sixth child is 19: Granny is 58 (39 + 19)
136. Solution:
The left number multiplied by 5 gives the total of the other two:
27 × 5 = 56 + 79
23 × 5 = 88 + 27
16 × 5 = 56 + ??
So ?? = 16 × 5 - 56 = 80 - 56 = 24
137. Solution:
 
 
The product of the two largest
minus the square of the smallest,
So the missing number is 45-16 = 29
138.
139. Solution:
Divide the 9 balls into 3 groups of 3. Compare the weight of two of those groups.
The heavier group should then be obvious, it will either tip the scales, or, if the scales stay balanced,
then it is the group you didn't include.
Now, choose 2 balls from this group and compare their weights, and using the same logic as before,
the heavier ball will be obvious.
140. Solution:
One possible method follows:
1. Fill up the 5 litre bucket
2. Pour the 5 litre bucket into the 3 litre bucket until the 3 litre bucket is full.
 
 
3. Empty the 3 litre bucket.
4. Pour the remaining 2 litres from the 5 litre bucket into the 3 litre bucket.
5. Fill up the 5 litre bucket again.

Weitere ähnliche Inhalte

Was ist angesagt?

9th class sample paper2015
9th class sample paper20159th class sample paper2015
9th class sample paper2015APEX INSTITUTE
 
Percentages average r&p
Percentages average r&pPercentages average r&p
Percentages average r&pSreeram Madhav
 
Class 9 maths value based
Class 9 maths value basedClass 9 maths value based
Class 9 maths value basednitishguptamaps
 
Class 9 Cbse Maths Sample Paper Model 1
Class 9 Cbse Maths Sample Paper Model 1Class 9 Cbse Maths Sample Paper Model 1
Class 9 Cbse Maths Sample Paper Model 1Sunaina Rawat
 
Ch 2014-solved
Ch 2014-solvedCh 2014-solved
Ch 2014-solveddrtm316
 
Mathematics model exam_vol-2_ppt_design
Mathematics model exam_vol-2_ppt_designMathematics model exam_vol-2_ppt_design
Mathematics model exam_vol-2_ppt_designOshane Milligan
 
Math genni-sample-paper-class-8-for-4-year
Math genni-sample-paper-class-8-for-4-yearMath genni-sample-paper-class-8-for-4-year
Math genni-sample-paper-class-8-for-4-yearNaveenGupta421124
 
It maths y5 final year examination
It maths y5 final year  examinationIt maths y5 final year  examination
It maths y5 final year examinationGRACE THILA
 
Summative assessment 1 class-x 2014-15
Summative assessment 1 class-x 2014-15Summative assessment 1 class-x 2014-15
Summative assessment 1 class-x 2014-15APEX INSTITUTE
 
Shortcuts in Mathematics for CAT, CET, GRE, GMAT or any similar competitive ...
Shortcuts in  Mathematics for CAT, CET, GRE, GMAT or any similar competitive ...Shortcuts in  Mathematics for CAT, CET, GRE, GMAT or any similar competitive ...
Shortcuts in Mathematics for CAT, CET, GRE, GMAT or any similar competitive ...paijayant
 
Class 9 Cbse Maths Question Paper
Class 9 Cbse Maths Question PaperClass 9 Cbse Maths Question Paper
Class 9 Cbse Maths Question PaperSunaina Rawat
 
Soal latihan tentang himpunan
Soal latihan tentang himpunanSoal latihan tentang himpunan
Soal latihan tentang himpunanmdimasv22
 
Class 9 Cbse Maths Sample Paper Term 2 Model 1
Class 9 Cbse Maths Sample Paper Term 2 Model 1Class 9 Cbse Maths Sample Paper Term 2 Model 1
Class 9 Cbse Maths Sample Paper Term 2 Model 1Sunaina Rawat
 

Was ist angesagt? (17)

9th class sample paper2015
9th class sample paper20159th class sample paper2015
9th class sample paper2015
 
Percentages average r&p
Percentages average r&pPercentages average r&p
Percentages average r&p
 
Class 9 maths value based
Class 9 maths value basedClass 9 maths value based
Class 9 maths value based
 
Numerical reasoning II
Numerical reasoning IINumerical reasoning II
Numerical reasoning II
 
Class 9 Cbse Maths Sample Paper Model 1
Class 9 Cbse Maths Sample Paper Model 1Class 9 Cbse Maths Sample Paper Model 1
Class 9 Cbse Maths Sample Paper Model 1
 
Ch 2014-solved
Ch 2014-solvedCh 2014-solved
Ch 2014-solved
 
Qb maths
Qb mathsQb maths
Qb maths
 
Mathematics model exam_vol-2_ppt_design
Mathematics model exam_vol-2_ppt_designMathematics model exam_vol-2_ppt_design
Mathematics model exam_vol-2_ppt_design
 
Math genni-sample-paper-class-8-for-4-year
Math genni-sample-paper-class-8-for-4-yearMath genni-sample-paper-class-8-for-4-year
Math genni-sample-paper-class-8-for-4-year
 
It maths y5 final year examination
It maths y5 final year  examinationIt maths y5 final year  examination
It maths y5 final year examination
 
Kertas model pep sebenar (pmr)
Kertas model pep sebenar (pmr)Kertas model pep sebenar (pmr)
Kertas model pep sebenar (pmr)
 
Summative assessment 1 class-x 2014-15
Summative assessment 1 class-x 2014-15Summative assessment 1 class-x 2014-15
Summative assessment 1 class-x 2014-15
 
Shortcuts in Mathematics for CAT, CET, GRE, GMAT or any similar competitive ...
Shortcuts in  Mathematics for CAT, CET, GRE, GMAT or any similar competitive ...Shortcuts in  Mathematics for CAT, CET, GRE, GMAT or any similar competitive ...
Shortcuts in Mathematics for CAT, CET, GRE, GMAT or any similar competitive ...
 
Class 9 Cbse Maths Question Paper
Class 9 Cbse Maths Question PaperClass 9 Cbse Maths Question Paper
Class 9 Cbse Maths Question Paper
 
Soal latihan tentang himpunan
Soal latihan tentang himpunanSoal latihan tentang himpunan
Soal latihan tentang himpunan
 
Letmath
LetmathLetmath
Letmath
 
Class 9 Cbse Maths Sample Paper Term 2 Model 1
Class 9 Cbse Maths Sample Paper Term 2 Model 1Class 9 Cbse Maths Sample Paper Term 2 Model 1
Class 9 Cbse Maths Sample Paper Term 2 Model 1
 

Ähnlich wie Mental maths ix q & a

Actuarial Science (ACET) Mock Test Paper III By Sourav Sir's Classes
Actuarial Science (ACET) Mock Test Paper III By Sourav Sir's ClassesActuarial Science (ACET) Mock Test Paper III By Sourav Sir's Classes
Actuarial Science (ACET) Mock Test Paper III By Sourav Sir's ClassesSOURAV DAS
 
ACTUARIAL SCIENCE ACET MODEL PAPER WITH SOLUTION BY SOURAV SIR'S CLASSES 983...
ACTUARIAL SCIENCE ACET MODEL PAPER WITH SOLUTION BY SOURAV SIR'S CLASSES  983...ACTUARIAL SCIENCE ACET MODEL PAPER WITH SOLUTION BY SOURAV SIR'S CLASSES  983...
ACTUARIAL SCIENCE ACET MODEL PAPER WITH SOLUTION BY SOURAV SIR'S CLASSES 983...SOURAV DAS
 
ACTUARIAL SCIENCE | MODEL TEST EXAM | NOTES SOLUTIONS STUDY MATERIAL BY SOURA...
ACTUARIAL SCIENCE | MODEL TEST EXAM | NOTES SOLUTIONS STUDY MATERIAL BY SOURA...ACTUARIAL SCIENCE | MODEL TEST EXAM | NOTES SOLUTIONS STUDY MATERIAL BY SOURA...
ACTUARIAL SCIENCE | MODEL TEST EXAM | NOTES SOLUTIONS STUDY MATERIAL BY SOURA...SOURAV DAS
 
Clock Aptitude Questions and Answers | Clock Aptitude Problem | JobzStore
Clock Aptitude Questions and Answers | Clock Aptitude Problem | JobzStoreClock Aptitude Questions and Answers | Clock Aptitude Problem | JobzStore
Clock Aptitude Questions and Answers | Clock Aptitude Problem | JobzStoreashwamegh patil
 
Clock Aptitude Questions and Answers | Clock Aptitude Problem | JobzStore
Clock Aptitude Questions and Answers | Clock Aptitude Problem | JobzStoreClock Aptitude Questions and Answers | Clock Aptitude Problem | JobzStore
Clock Aptitude Questions and Answers | Clock Aptitude Problem | JobzStoreashwamegh patil
 
Practice Questions In Quantitative Aptitude
Practice Questions In Quantitative AptitudePractice Questions In Quantitative Aptitude
Practice Questions In Quantitative AptitudeDr. Trilok Kumar Jain
 
Diagnostic test (math vi)
Diagnostic test (math vi)Diagnostic test (math vi)
Diagnostic test (math vi)rebecca031787
 
Guide de problem solving avec solution - préparation entretiens conseil
Guide de problem solving  avec solution - préparation entretiens conseilGuide de problem solving  avec solution - préparation entretiens conseil
Guide de problem solving avec solution - préparation entretiens conseilJocelyne Giambiaggi
 
GEN ED MATH 2 PNU[1].pptx
GEN ED MATH 2 PNU[1].pptxGEN ED MATH 2 PNU[1].pptx
GEN ED MATH 2 PNU[1].pptxAldineRaytan
 
Mathematics Specialization.pptx
Mathematics Specialization.pptxMathematics Specialization.pptx
Mathematics Specialization.pptxRonnalynAranda2
 
GRADE 5 SESSION 5.pptx
GRADE 5 SESSION 5.pptxGRADE 5 SESSION 5.pptx
GRADE 5 SESSION 5.pptxLuisSalenga1
 
Units 1 3 review
Units 1 3 reviewUnits 1 3 review
Units 1 3 reviewmlabuski
 

Ähnlich wie Mental maths ix q & a (20)

Numerical Ability.pptx
Numerical Ability.pptxNumerical Ability.pptx
Numerical Ability.pptx
 
Naren Quiz 14 Dr.K.Karthikeyan
Naren Quiz 14 Dr.K.KarthikeyanNaren Quiz 14 Dr.K.Karthikeyan
Naren Quiz 14 Dr.K.Karthikeyan
 
GenEd_math.pdf
GenEd_math.pdfGenEd_math.pdf
GenEd_math.pdf
 
Actuarial Science (ACET) Mock Test Paper III By Sourav Sir's Classes
Actuarial Science (ACET) Mock Test Paper III By Sourav Sir's ClassesActuarial Science (ACET) Mock Test Paper III By Sourav Sir's Classes
Actuarial Science (ACET) Mock Test Paper III By Sourav Sir's Classes
 
ACTUARIAL SCIENCE ACET MODEL PAPER WITH SOLUTION BY SOURAV SIR'S CLASSES 983...
ACTUARIAL SCIENCE ACET MODEL PAPER WITH SOLUTION BY SOURAV SIR'S CLASSES  983...ACTUARIAL SCIENCE ACET MODEL PAPER WITH SOLUTION BY SOURAV SIR'S CLASSES  983...
ACTUARIAL SCIENCE ACET MODEL PAPER WITH SOLUTION BY SOURAV SIR'S CLASSES 983...
 
ACTUARIAL SCIENCE | MODEL TEST EXAM | NOTES SOLUTIONS STUDY MATERIAL BY SOURA...
ACTUARIAL SCIENCE | MODEL TEST EXAM | NOTES SOLUTIONS STUDY MATERIAL BY SOURA...ACTUARIAL SCIENCE | MODEL TEST EXAM | NOTES SOLUTIONS STUDY MATERIAL BY SOURA...
ACTUARIAL SCIENCE | MODEL TEST EXAM | NOTES SOLUTIONS STUDY MATERIAL BY SOURA...
 
Clock Aptitude Questions and Answers | Clock Aptitude Problem | JobzStore
Clock Aptitude Questions and Answers | Clock Aptitude Problem | JobzStoreClock Aptitude Questions and Answers | Clock Aptitude Problem | JobzStore
Clock Aptitude Questions and Answers | Clock Aptitude Problem | JobzStore
 
Clock Aptitude Questions and Answers | Clock Aptitude Problem | JobzStore
Clock Aptitude Questions and Answers | Clock Aptitude Problem | JobzStoreClock Aptitude Questions and Answers | Clock Aptitude Problem | JobzStore
Clock Aptitude Questions and Answers | Clock Aptitude Problem | JobzStore
 
Naren Quiz 10 Dr.K.Karthikeyan
Naren Quiz 10 Dr.K.KarthikeyanNaren Quiz 10 Dr.K.Karthikeyan
Naren Quiz 10 Dr.K.Karthikeyan
 
Unit 3 - Averages.pptx
Unit 3 - Averages.pptxUnit 3 - Averages.pptx
Unit 3 - Averages.pptx
 
Practice Questions In Quantitative Aptitude
Practice Questions In Quantitative AptitudePractice Questions In Quantitative Aptitude
Practice Questions In Quantitative Aptitude
 
file6192075727593843027.pptx
file6192075727593843027.pptxfile6192075727593843027.pptx
file6192075727593843027.pptx
 
Apttitude
ApttitudeApttitude
Apttitude
 
Diagnostic test (math vi)
Diagnostic test (math vi)Diagnostic test (math vi)
Diagnostic test (math vi)
 
Guide de problem solving avec solution - préparation entretiens conseil
Guide de problem solving  avec solution - préparation entretiens conseilGuide de problem solving  avec solution - préparation entretiens conseil
Guide de problem solving avec solution - préparation entretiens conseil
 
GEN ED MATH 2 PNU[1].pptx
GEN ED MATH 2 PNU[1].pptxGEN ED MATH 2 PNU[1].pptx
GEN ED MATH 2 PNU[1].pptx
 
Mathematics Specialization.pptx
Mathematics Specialization.pptxMathematics Specialization.pptx
Mathematics Specialization.pptx
 
Assignment quant
Assignment quantAssignment quant
Assignment quant
 
GRADE 5 SESSION 5.pptx
GRADE 5 SESSION 5.pptxGRADE 5 SESSION 5.pptx
GRADE 5 SESSION 5.pptx
 
Units 1 3 review
Units 1 3 reviewUnits 1 3 review
Units 1 3 review
 

Kürzlich hochgeladen

Call Us ➥9911191017▻Young Call Girls In Guru Dronacharya Metro Station Delhi NCR
Call Us ➥9911191017▻Young Call Girls In Guru Dronacharya Metro Station Delhi NCRCall Us ➥9911191017▻Young Call Girls In Guru Dronacharya Metro Station Delhi NCR
Call Us ➥9911191017▻Young Call Girls In Guru Dronacharya Metro Station Delhi NCRsafdarjungdelhi1
 
Call Us ☎97110√14705🔝 Call Girls in Majnu Ka Tilla (Delhi NCR)
Call Us ☎97110√14705🔝 Call Girls in Majnu Ka Tilla (Delhi NCR)Call Us ☎97110√14705🔝 Call Girls in Majnu Ka Tilla (Delhi NCR)
Call Us ☎97110√14705🔝 Call Girls in Majnu Ka Tilla (Delhi NCR)thapagita
 
(9818099198) Noida Escorts Service Sector 60 (NOIDA CALL GIRLS)
(9818099198) Noida Escorts Service Sector 60 (NOIDA CALL GIRLS)(9818099198) Noida Escorts Service Sector 60 (NOIDA CALL GIRLS)
(9818099198) Noida Escorts Service Sector 60 (NOIDA CALL GIRLS)riyaescorts54
 
(9818099198) Call Girls In Noida Sector 88 (NOIDA ESCORTS)
(9818099198) Call Girls In Noida Sector 88 (NOIDA ESCORTS)(9818099198) Call Girls In Noida Sector 88 (NOIDA ESCORTS)
(9818099198) Call Girls In Noida Sector 88 (NOIDA ESCORTS)riyaescorts54
 
9643097474 Full Enjoy @24/7 Call Girls in Saket Metro Delhi NCR
9643097474 Full Enjoy @24/7 Call Girls in Saket Metro Delhi NCR9643097474 Full Enjoy @24/7 Call Girls in Saket Metro Delhi NCR
9643097474 Full Enjoy @24/7 Call Girls in Saket Metro Delhi NCRthapariya601
 
Call Girls In saket 9711800081 Low Rate Short 1500 Night ...
Call Girls In saket 9711800081 Low Rate Short 1500 Night ...Call Girls In saket 9711800081 Low Rate Short 1500 Night ...
Call Girls In saket 9711800081 Low Rate Short 1500 Night ...gitathapa4
 
9643097474 Full Enjoy @24/7 Call Girls In Khirki Extension Delhi Ncr
9643097474 Full Enjoy @24/7 Call Girls In Khirki Extension Delhi Ncr9643097474 Full Enjoy @24/7 Call Girls In Khirki Extension Delhi Ncr
9643097474 Full Enjoy @24/7 Call Girls In Khirki Extension Delhi Ncrthapariya601
 
Call Girls In New Delhi Railway Station 9667422720 Top Quality Escorts Service
Call Girls In New Delhi Railway Station 9667422720 Top Quality Escorts ServiceCall Girls In New Delhi Railway Station 9667422720 Top Quality Escorts Service
Call Girls In New Delhi Railway Station 9667422720 Top Quality Escorts ServiceLipikasharma29
 
Justdial Call Girls In Vaishali, Ghaziabad 8800357707 Escorts Service
Justdial Call Girls In Vaishali, Ghaziabad 8800357707 Escorts ServiceJustdial Call Girls In Vaishali, Ghaziabad 8800357707 Escorts Service
Justdial Call Girls In Vaishali, Ghaziabad 8800357707 Escorts Servicemonikaservice1
 
9643097474 Full Enjoy @24/7 Call Girls In Munirka Delhi Ncr
9643097474 Full Enjoy @24/7 Call Girls In Munirka Delhi Ncr9643097474 Full Enjoy @24/7 Call Girls In Munirka Delhi Ncr
9643097474 Full Enjoy @24/7 Call Girls In Munirka Delhi Ncrthapariya601
 
▶ ●─Hookup Call Girls In Noida Sector 137 (Noida) ⎝9667422720⎠ Delhi Female E...
▶ ●─Hookup Call Girls In Noida Sector 137 (Noida) ⎝9667422720⎠ Delhi Female E...▶ ●─Hookup Call Girls In Noida Sector 137 (Noida) ⎝9667422720⎠ Delhi Female E...
▶ ●─Hookup Call Girls In Noida Sector 137 (Noida) ⎝9667422720⎠ Delhi Female E...Lipikasharma29
 
Call Girls In Goa North Goa 9899855202 Direct Cash 0nline Payment For Genuine
Call Girls In Goa North Goa 9899855202 Direct Cash 0nline Payment For GenuineCall Girls In Goa North Goa 9899855202 Direct Cash 0nline Payment For Genuine
Call Girls In Goa North Goa 9899855202 Direct Cash 0nline Payment For Genuinedelhincr993
 
9643097474 Full Enjoy @24/7 Call Girls In Mahipalpur Delhi Ncr
9643097474 Full Enjoy @24/7 Call Girls In Mahipalpur Delhi Ncr9643097474 Full Enjoy @24/7 Call Girls In Mahipalpur Delhi Ncr
9643097474 Full Enjoy @24/7 Call Girls In Mahipalpur Delhi Ncrthapariya601
 
Trusted Call~Girls In Rohini Delhi꧁❤ 9667422720 ❤꧂Escorts
Trusted Call~Girls In Rohini Delhi꧁❤ 9667422720 ❤꧂EscortsTrusted Call~Girls In Rohini Delhi꧁❤ 9667422720 ❤꧂Escorts
Trusted Call~Girls In Rohini Delhi꧁❤ 9667422720 ❤꧂EscortsLipikasharma29
 
Justdial Call Girls In Moolchand Metro Delhi 9911191017 Escorts Service
Justdial Call Girls In Moolchand Metro Delhi 9911191017 Escorts ServiceJustdial Call Girls In Moolchand Metro Delhi 9911191017 Escorts Service
Justdial Call Girls In Moolchand Metro Delhi 9911191017 Escorts Servicesafdarjungdelhi1
 
Call Girls In Munirka,( Delhi — 9667422720 Escorts ) Service
Call Girls In Munirka,( Delhi — 9667422720 Escorts ) ServiceCall Girls In Munirka,( Delhi — 9667422720 Escorts ) Service
Call Girls In Munirka,( Delhi — 9667422720 Escorts ) ServiceLipikasharma29
 
FULL ENJOY Call Girls In Gurgaon Call 8588836666 Escorts Service
FULL ENJOY Call Girls In Gurgaon  Call 8588836666 Escorts ServiceFULL ENJOY Call Girls In Gurgaon  Call 8588836666 Escorts Service
FULL ENJOY Call Girls In Gurgaon Call 8588836666 Escorts ServiceCALLGIRLS DELHI
 
9911558886 Cash on Hand Low Costly Russian Call Girls In Naraina Vihar
9911558886 Cash on Hand Low Costly Russian Call Girls In Naraina Vihar9911558886 Cash on Hand Low Costly Russian Call Girls In Naraina Vihar
9911558886 Cash on Hand Low Costly Russian Call Girls In Naraina Viharmalikasharmakk1
 
Call Us ≽ 9643900018 ≼ Call Girls In Lado Sarai (Delhi)
Call Us ≽ 9643900018 ≼ Call Girls In Lado Sarai (Delhi)Call Us ≽ 9643900018 ≼ Call Girls In Lado Sarai (Delhi)
Call Us ≽ 9643900018 ≼ Call Girls In Lado Sarai (Delhi)ayushiverma1100
 

Kürzlich hochgeladen (20)

Call Us ➥9911191017▻Young Call Girls In Guru Dronacharya Metro Station Delhi NCR
Call Us ➥9911191017▻Young Call Girls In Guru Dronacharya Metro Station Delhi NCRCall Us ➥9911191017▻Young Call Girls In Guru Dronacharya Metro Station Delhi NCR
Call Us ➥9911191017▻Young Call Girls In Guru Dronacharya Metro Station Delhi NCR
 
Call Us ☎97110√14705🔝 Call Girls in Majnu Ka Tilla (Delhi NCR)
Call Us ☎97110√14705🔝 Call Girls in Majnu Ka Tilla (Delhi NCR)Call Us ☎97110√14705🔝 Call Girls in Majnu Ka Tilla (Delhi NCR)
Call Us ☎97110√14705🔝 Call Girls in Majnu Ka Tilla (Delhi NCR)
 
(9818099198) Noida Escorts Service Sector 60 (NOIDA CALL GIRLS)
(9818099198) Noida Escorts Service Sector 60 (NOIDA CALL GIRLS)(9818099198) Noida Escorts Service Sector 60 (NOIDA CALL GIRLS)
(9818099198) Noida Escorts Service Sector 60 (NOIDA CALL GIRLS)
 
(9818099198) Call Girls In Noida Sector 88 (NOIDA ESCORTS)
(9818099198) Call Girls In Noida Sector 88 (NOIDA ESCORTS)(9818099198) Call Girls In Noida Sector 88 (NOIDA ESCORTS)
(9818099198) Call Girls In Noida Sector 88 (NOIDA ESCORTS)
 
9643097474 Full Enjoy @24/7 Call Girls in Saket Metro Delhi NCR
9643097474 Full Enjoy @24/7 Call Girls in Saket Metro Delhi NCR9643097474 Full Enjoy @24/7 Call Girls in Saket Metro Delhi NCR
9643097474 Full Enjoy @24/7 Call Girls in Saket Metro Delhi NCR
 
Call Girls In saket 9711800081 Low Rate Short 1500 Night ...
Call Girls In saket 9711800081 Low Rate Short 1500 Night ...Call Girls In saket 9711800081 Low Rate Short 1500 Night ...
Call Girls In saket 9711800081 Low Rate Short 1500 Night ...
 
9643097474 Full Enjoy @24/7 Call Girls In Khirki Extension Delhi Ncr
9643097474 Full Enjoy @24/7 Call Girls In Khirki Extension Delhi Ncr9643097474 Full Enjoy @24/7 Call Girls In Khirki Extension Delhi Ncr
9643097474 Full Enjoy @24/7 Call Girls In Khirki Extension Delhi Ncr
 
9953056974 Low Rate Call Girls Delhi NCR
9953056974 Low Rate Call Girls Delhi NCR9953056974 Low Rate Call Girls Delhi NCR
9953056974 Low Rate Call Girls Delhi NCR
 
Call Girls In New Delhi Railway Station 9667422720 Top Quality Escorts Service
Call Girls In New Delhi Railway Station 9667422720 Top Quality Escorts ServiceCall Girls In New Delhi Railway Station 9667422720 Top Quality Escorts Service
Call Girls In New Delhi Railway Station 9667422720 Top Quality Escorts Service
 
Justdial Call Girls In Vaishali, Ghaziabad 8800357707 Escorts Service
Justdial Call Girls In Vaishali, Ghaziabad 8800357707 Escorts ServiceJustdial Call Girls In Vaishali, Ghaziabad 8800357707 Escorts Service
Justdial Call Girls In Vaishali, Ghaziabad 8800357707 Escorts Service
 
9643097474 Full Enjoy @24/7 Call Girls In Munirka Delhi Ncr
9643097474 Full Enjoy @24/7 Call Girls In Munirka Delhi Ncr9643097474 Full Enjoy @24/7 Call Girls In Munirka Delhi Ncr
9643097474 Full Enjoy @24/7 Call Girls In Munirka Delhi Ncr
 
▶ ●─Hookup Call Girls In Noida Sector 137 (Noida) ⎝9667422720⎠ Delhi Female E...
▶ ●─Hookup Call Girls In Noida Sector 137 (Noida) ⎝9667422720⎠ Delhi Female E...▶ ●─Hookup Call Girls In Noida Sector 137 (Noida) ⎝9667422720⎠ Delhi Female E...
▶ ●─Hookup Call Girls In Noida Sector 137 (Noida) ⎝9667422720⎠ Delhi Female E...
 
Call Girls In Goa North Goa 9899855202 Direct Cash 0nline Payment For Genuine
Call Girls In Goa North Goa 9899855202 Direct Cash 0nline Payment For GenuineCall Girls In Goa North Goa 9899855202 Direct Cash 0nline Payment For Genuine
Call Girls In Goa North Goa 9899855202 Direct Cash 0nline Payment For Genuine
 
9643097474 Full Enjoy @24/7 Call Girls In Mahipalpur Delhi Ncr
9643097474 Full Enjoy @24/7 Call Girls In Mahipalpur Delhi Ncr9643097474 Full Enjoy @24/7 Call Girls In Mahipalpur Delhi Ncr
9643097474 Full Enjoy @24/7 Call Girls In Mahipalpur Delhi Ncr
 
Trusted Call~Girls In Rohini Delhi꧁❤ 9667422720 ❤꧂Escorts
Trusted Call~Girls In Rohini Delhi꧁❤ 9667422720 ❤꧂EscortsTrusted Call~Girls In Rohini Delhi꧁❤ 9667422720 ❤꧂Escorts
Trusted Call~Girls In Rohini Delhi꧁❤ 9667422720 ❤꧂Escorts
 
Justdial Call Girls In Moolchand Metro Delhi 9911191017 Escorts Service
Justdial Call Girls In Moolchand Metro Delhi 9911191017 Escorts ServiceJustdial Call Girls In Moolchand Metro Delhi 9911191017 Escorts Service
Justdial Call Girls In Moolchand Metro Delhi 9911191017 Escorts Service
 
Call Girls In Munirka,( Delhi — 9667422720 Escorts ) Service
Call Girls In Munirka,( Delhi — 9667422720 Escorts ) ServiceCall Girls In Munirka,( Delhi — 9667422720 Escorts ) Service
Call Girls In Munirka,( Delhi — 9667422720 Escorts ) Service
 
FULL ENJOY Call Girls In Gurgaon Call 8588836666 Escorts Service
FULL ENJOY Call Girls In Gurgaon  Call 8588836666 Escorts ServiceFULL ENJOY Call Girls In Gurgaon  Call 8588836666 Escorts Service
FULL ENJOY Call Girls In Gurgaon Call 8588836666 Escorts Service
 
9911558886 Cash on Hand Low Costly Russian Call Girls In Naraina Vihar
9911558886 Cash on Hand Low Costly Russian Call Girls In Naraina Vihar9911558886 Cash on Hand Low Costly Russian Call Girls In Naraina Vihar
9911558886 Cash on Hand Low Costly Russian Call Girls In Naraina Vihar
 
Call Us ≽ 9643900018 ≼ Call Girls In Lado Sarai (Delhi)
Call Us ≽ 9643900018 ≼ Call Girls In Lado Sarai (Delhi)Call Us ≽ 9643900018 ≼ Call Girls In Lado Sarai (Delhi)
Call Us ≽ 9643900018 ≼ Call Girls In Lado Sarai (Delhi)
 

Mental maths ix q & a

  • 1.     APTITUDE: Time and Work 1. A can do a work in 15 days and B in 20 days. If they work on it together for 4 days, then the fraction of the work that is left is : A. 1 4 B. 1 10 C. 7 15 D. 8 15 Answer: Option D Explanation: A's 1 day's work = 1 ; 15 B's 1 day's work = 1 ; 20 (A + B)'s 1 day's work = 1 + 1 = 7 . 15 20 60 (A + B)'s 4 day's work = 7 x 4 = 7 . 60 15 Therefore, Remaining work = 1 - 7 = 8 . 15 15 2. A can lay railway track between two given stations in 16 days and B can do the same job in 12 days. With help of C, they did the job in 4 days only. Then, C alone can do the job in: A. 9 1 days 5 B. 9 2 days 5 C. 9 3 days 5 D. 10 Answer: Option C Explanation:(A + B + C)'s 1 day's work = 1 , 4
  • 2.     A's 1 day's work = 1 , 16 B's 1 day's work = 1 . 12 C's 1 day's work = 1 - 1 + 1 = 1 - 7 = 5 . 4 16 12 4 48 48 So, C alone can do the work in 48 = 9 3 days. 5 5 3. A can do a piece of work in 4 hours; B and C together can do it in 3 hours, while A and C together can do it in 2 hours. How long will B alone take to do it? A. 8 hours B. 10 hours C. 12 hours D. 24 hours Answer: Option C Explanation: A's 1 hour's work = 1 ; 4 (B + C)'s 1 hour's work = 1 ; 3 (A + C)'s 1 hour's work = 1 . 2 (A + B + C)'s 1 hour's work = 1 + 1 = 7 . 4 3 12 B's 1 hour's work = 7 - 1 = 1 . 12 2 12 B alone will take 12 hours to do the work. 4. If 6 men and 8 boys can do a piece of work in 10 days while 26 men and 48 boys can do the same in 2 days, the time taken by 15 men and 20 boys in doing the same type of work will be: A. 4 days B. 5 days C. 6 days D. 7 days
  • 3.     Answer: Option A Explanation: Let 1 man's 1 day's work = x and 1 boy's 1 day's work = y. Then, 6x + 8y = 1 and 26x + 48y = 1 . 10 2 Solving these two equations, we get : x = 1 and y = 1 . 100 200 (15 men + 20 boy)'s 1 day's work = 15 + 20 = 1 . 100 200 4 15 men and 20 boys can do the work in 4 days. -------------------------------------------------------- Time and Distance 5. An aeroplane covers a certain distance at a speed of 240 kmph in 5 hours. To cover the same distance in 1 hours, it must travel at a speed of: A. 300 kmph B. 360 kmph C. 600 kmph D. 720 kmph Answer: Option D Explanation: Distance = (240 x 5) = 1200 km. Speed = Distance/Time Speed = 1200/(5/3) km/hr. [We can write 1 hours as 5/3 hours] Required speed = 1200 x 3 km/hr = 720 km/hr. 5
  • 4.     6. If a person walks at 14 km/hr instead of 10 km/hr, he would have walked 20 km more. The actual distance travelled by him is: A. 50 km B. 56 km C. 70 km D. 80 km Answer: Option A Explanation: Let the actual distance travelled be x km. Then, x = x + 20 10 14 14x = 10x + 200 4x = 200 x = 50 km. Simple Interest 7. A sum of Rs. 12,500 amounts to Rs. 15,500 in 4 years at the rate of simple interest. What is the rate of interest? A. 3% B. 4% C. 5% D. 6% E. None of these Answer: Option D
  • 5.     Explanation: S.I. = Rs. (15500 - 12500) = Rs. 3000. Rate = 100 x 3000 % = 6% 12500 x 4 8. A sum of money at simple interest amounts to Rs. 815 in 3 years and to Rs. 854 in 4 years. The sum is: A. Rs. 650 B. Rs. 690 C. Rs. 698 D. Rs. 700 Answer: Option C Explanation: S.I. for 1 year = Rs. (854 - 815) = Rs. 39. S.I. for 3 years = Rs.(39 x 3) = Rs. 117. Principal = Rs. (815 - 117) = Rs. 698. 9. A man took loan from a bank at the rate of 12% p.a. simple interest. After 3 years he had to pay Rs. 5400 interest only for the period. The principal amount borrowed by him was: A. Rs. 2000 B. Rs. 10,000 C. Rs. 15,000 D. Rs. 20,000 Answer: Option C Explanation: Principal = Rs. 100 x 5400 = Rs. 15000. 12 x 3
  • 6.     10. What will be the ratio of simple interest earned by certain amount at the same rate of interest for 6 years and that for 9 years? A. 1 : 3 B. 1 : 4 C. 2 : 3 D. Data inadequate E. None of these Answer: Option C Explanation: Let the principal be P and rate of interest be R%. Required ratio = P x R x 6 100 = 6PR = 6 = 2 : 3. P x R x 9 100 9PR 9 Profit and Loss 11. Alfred buys an old scooter for Rs. 4700 and spends Rs. 800 on its repairs. If he sells the scooter for Rs. 5800, his gain percent is: A. 4 4 % 7 B. 5 5 % 11 C. 10% D. 12% Answer: Option B Explanation: Cost Price (C.P.) = Rs. (4700 + 800) = Rs. 5500. Selling Price (S.P.) = Rs. 5800.
  • 7.     Gain = (S.P.) - (C.P.) = Rs.(5800 - 5500) = Rs. 300. Gain % = 300 x 100 % = 5 5 % 5500 11 12. If selling price is doubled, the profit triples. Find the profit percent. Answer: Option B Explanation: Let C.P. be Rs. x and S.P. be Rs. y. Then, 3(y - x) = (2y - x) y = 2x. Profit = Rs. (y - x) = Rs. (2x - x) = Rs. x. Profit % = x x 100 % = 100%x 13. A vendor bought toffees at 6 for a rupee. How many for a rupee must he sell to gain 20%? A. 3 B. 4 C. 5 D. 6 Answer: Option C Explanation: C.P. of 6 toffees = Re. 1 S.P. of 6 toffees = 120% of Re. 1 = Rs. 6 5 For Rs. 6 , toffees sold = 6. 5 For Re. 1, toffees sold = 6 x 5 = 5. 6
  • 8.     14. A man buys a cycle for Rs. 1400 and sells it at a loss of 15%. What is the selling price of the cycle? A. Rs. 1090 B. Rs. 1160 C. Rs. 1190 D. Rs. 1202 Answer: Option C Explanation: S.P. = 85% of Rs. 1400 = Rs. 85 x 1400 = Rs. 1190 100 Percentage 15. A batsman scored 110 runs which included 3 boundaries and 8 sixes. What percent of his total score did he make by running between the wickets? A. 45% B. 45 5 % 11 C. 54 6 % 11 D. 55% Answer: Option B Explanation: Number of runs made by running = 110 - (3 x 4 + 8 x 6) = 110 - (60) = 50. Required percentage = 50 x 100 % = 45 5 % 110 11
  • 9.     16. If 20% of a = b, then b% of 20 is the same as: A. 4% of a B. 5% of a C. 20% of a D. None of these Answer: Option A Explanation: 20% of a = b 20 a = b. 100 b% of 20 = b x 20 = 20 a x 1 x 20 = 4 a = 4% of a. 100 100 100 100 17. In an election between two candidates, one got 55% of the total valid votes, 20% of the votes were invalid. If the total number of votes was 7500, the number of valid votes that the other candidate got, was: A. 2700 B. 2900 C. 3000 D. 3100 Answer: Option A Explanation: Number of valid votes = 80% of 7500 = 6000. Valid votes polled by other candidate = 45% of 6000 = 45 x 6000 = 2700. 100 18. Three candidates contested an election and received 1136, 7636 and 11628 votes respectively. What percentage of the total votes did the winning candidate get?
  • 10.     A. 57% B. 60% C. 65% D. 90% Answer: Option A Explanation: Total number of votes polled = (1136 + 7636 + 11628) = 20400. Required percentage = 11628 x 100 % = 57%.20400 19. The population of a town increased from 1,75,000 to 2,62,500 in a decade. The average percent increase of population per year is: A. 4.37% B. 5% C. 6% D. 8.75% Answer: Option B Explanation: Increase in 10 years = (262500 - 175000) = 87500. Increase% = 87500 x 100 % = 50%.175000 Required average = 50 % = 5%.10 Average 20. In the first 10 overs of a cricket game, the run rate was only 3.2. What should be the run rate in the remaining 40 overs to reach the target of 282 runs?
  • 11.     A. 6.25 B. 6.5 C. 6.75 D. 7 Answer: Option A Explanation: Required run rate = 282 - (3.2 x 10) = 250 = 6.25 40 40 21. A family consists of two grandparents, two parents and three grandchildren. The average age of the grandparents is 67 years, that of the parents is 35 years and that of the grandchildren is 6 years. What is the average age of the family? A. 28 4 years 7 B. 31 5 years 7 C. 32 1 years 7 D. None of these Answer: Option B Explanation: Required average = 67 x 2 + 35 x 2 + 6 x 3 2 + 2 + 3 = 134 + 70 + 18 7 = 222 7 = 31 5 years.
  • 12.     7 22. A grocer has a sale of Rs. 6435, Rs. 6927, Rs. 6855, Rs. 7230 and Rs. 6562 for 5 consecutive months. How much sale must he have in the sixth month so that he gets an average sale of Rs. 6500? A. Rs. 4991 B. Rs. 5991 C. Rs. 6001 D. Rs. 6991 Answer: Option A Explanation: Total sale for 5 months = Rs. (6435 + 6927 + 6855 + 7230 + 6562) = Rs. 34009. Required sale = Rs. [ (6500 x 6) - 34009 ] = Rs. (39000 - 34009) = Rs. 4991. 23. he average weight of 8 person's increases by 2.5 kg when a new person comes in place of one of them weighing 65 kg. What might be the weight of the new person? A. 76 kg B. 76.5 kg C. 85 kg D. Data inadequate E. None of these Answer: Option C Explanation: Total weight increased = (8 x 2.5) kg = 20 kg. Weight of new person = (65 + 20) kg = 85 kg.
  • 13.     24. The average age of husband, wife and their child 3 years ago was 27 years and that of wife and the child 5 years ago was 20 years. The present age of the husband is: A. 35 years B. 40 years C. 50 years D. None of these Answer: Option B Explanation: Sum of the present ages of husband, wife and child = (27 x 3 + 3 x 3) years = 90 years. Sum of the present ages of wife and child = (20 x 2 + 5 x 2) years = 50 years. Husband's present age = (90 - 50) years = 40 years. 25. The average weight of 16 boys in a class is 50.25 kg and that of the remaining 8 boys is 45.15 kg. Find the average weights of all the boys in the class. A. 47.55 kg B. 48 kg C. 48.55 kg D. 49.25 kg Answer: Option C Explanation: Required average = 50.25 x 16 + 45.15 x 8 16 + 8 = 804 + 361.20 24 = 1165.20 24 = 48.55
  • 14.     26. If the average marks of three batches of 55, 60 and 45 students respectively is 50, 55, 60, then the average marks of all the students is: A. 53.33 B. 54.68 C. 55 D. None of these Answer: Option B Explanation: Required average = 55 x 50 + 60 x 55 + 45 x 60 55 + 60 + 45 = 2750 + 3300 + 2700 160 = 8750 160 Decimal Fraction 27. Evaluate : (2.39)2 - (1.61)2 2.39 - 1.61 A. 2 B. 4 C. 6 D. 8 Answer: Option B Explanation: Given Expression = a2 - b2 = (a + b)(a - b) = (a + b) = (2.39 + 1.61) = 4. a - b (a - b)
  • 15.     28. What decimal of an hour is a second ? A. .0025 B. .0256 C. .00027 D. .000126 Answer: Option C Explanation: Required decimal = 1 = 1 = .00027 60 x 60 3600 29. The value of 0.1 x 0.1 x 0.1 + 0.02 x 0.02 x 0.02 is: 0.2 x 0.2 x 0.2 + 0.04 x 0.04 x 0.04 A. 0.0125 B. 0.125 C. 0.25 D. 0.5 Answer: Option B Explanation: Given expression = (0.1)3 + (0.02)3 = 1 = 0.125 23 [(0.1)3 + (0.02)3 ] 8 30. 4.2 x 4.2 - 1.9 x 1.9 is equal to: 2.3 x 6.1 A. 0.5 B. 1.0 C. 20 D. 22 Answer: Option B Explanation:
  • 16.     Given Expression = (a2 - b2 ) = (a2 - b2 ) = 1. (a + b)(a - b) (a2 - b2 ) 31. If 144 = 14.4 , then the value of x is: 0.144 x A. 0.0144 B. 1.44 C. 14.4 D. 144 Answer: Option A Explanation: 144 = 14.4 0.144 x 144 x 1000 = 14.4 144 x x = 14.4 = 0.0144 1000 LOGICAL REASONING: Number Series 32. Look at this series: 2, 1, (1/2), (1/4), ... What number should come next? A. (1/3) B. (1/8) C. (2/8) D. (1/16)
  • 17.     Answer: Option B Explanation: This is a simple division series; each number is one-half of the previous number. In other terms to say, the number is divided by 2 successively to get the next result. 4/2 = 2 2/2 = 1 1/2 = 1/2 (1/2)/2 = 1/4 (1/4)/2 = 1/8 and so on. 33. Look at this series: 58, 52, 46, 40, 34, ... What number should come next? A. 26 B. 28 C. 30 D. 32 Answer: Option B Explanation: This is a simple subtraction series. Each number is 6 less than the previous number. 34. Look at this series: 3, 4, 7, 8, 11, 12, ... What number should come next? A. 7 B. 10 C. 14 D. 15 Answer: Option D Explanation: This alternating addition series begins with 3; then 1 is added to give 4; then 3 is added to give 7; then 1 is added, and so on.
  • 18.     35. Look at this series: 31, 29, 24, 22, 17, ... What number should come next? A. 15 B. 14 C. 13 D. 12 Answer: Option A Explanation: This is a simple alternating subtraction series, which subtracts 2, then 5. DATA INTERPRETATION 36. Bar Charts 1. What is the average sales of all the branches (in thousand numbers) for the year 2000? A. 73 B. 80 C. 83 D. 88
  • 19.     Answer: Option B Explanation: Average sales of all the six branches (in thousand numbers) for the year 2000 = 1 x [80 + 75 + 95 + 85 + 75 + 70] 6 = 80. 2. Total sales of branches B1, B3 and B5 together for both the years (in thousand numbers) is? A. 250 B. 310 C. 435 D. 560 Answer: Option D Explanation: Total sales of branches B1, B3 and B5 for both the years (in thousand numbers) = (80 + 105) + (95 + 110) + (75 + 95) = 560. 3. What is the ratio of the total sales of branch B2 for both years to the total sales of branch B4 for both years? A. 2:3 B. 3:5 C. 4:5 D. 7:9 Answer: Option D Explanation:
  • 20.     Required ratio = (75 + 65) = 140 = 7 . (85 + 95) 180 9 NON VERBAL REASONING Series 37. Select a figure from amongst the Answer Figures which will continue the same series as established by the five Problem Figures. Problem Figures: Answer Figures: (A) (B) (C) (D) (E) (1) (2) (3) (4) (5) A. 1 B. 2 C. 3 D. 4 E. 5 Answer: Option A Explanation: In each step, the dot moves one space CW and the arrow moves two spaces CW. 38. Select a figure from amongst the Answer Figures which will continue the same series as established by the five Problem Figures. Problem Figures: Answer Figures: (A) (B) (C) (D) (E) (1) (2) (3) (4) (5) A. 1 B. 2 C. 3 D. 4 E. 5
  • 21.     Answer: Option C Explanation: The pin rotates 45o CW and 90o CW alternately and moves one space (each space is equal to half-a- side of the square) and two spaces CW alternately. The arrow rotates 90o ACW and 45o ACW alternately and moves two spaces and one space. 39. Select a figure from amongst the Answer Figures which will continue the same series as established by the five Problem Figures. Problem Figures: Answer Figures: (A) (B) (C) (D) (E) (1) (2) (3) (4) (5) A. 1 B. 2 C. 3 D. 4 E. 5 Answer: Option C Explanation: In each step, the pin rotates 90o CW and the arrow rotates 90o ACW. 40. Select a figure from amongst the Answer Figures which will continue the same series as established by the five Problem Figures. Problem Figures: Answer Figures: (A) (B) (C) (D) (E) (1) (2) (3) (4) (5) A. 1 B. 2
  • 22.     C. 3 D. 4 E. 5 Answer: Option D Explanation: In each step, all the elements move to the adjacent corner (of the square boundary) in a CW direction and the element that reaches the upper-left corner gets vertically inverted. 42. Select a figure from amongst the Answer Figures which will continue the same series as established by the five Problem Figures. Problem Figures: Answer Figures: (A) (B) (C) (D) (E) (1) (2) (3) (4) (5) A. 1 B. 2 C. 3 D. 4 E. 5 Answer: Option D Explanation: In each step, the first element moves to the third position and gets replaced by a new element; the second and the third elements move to the first and the second positions respectively and the entire figure rotates 90o CW. Analytical Reasoning 41. Find the number of triangles in the given figure. A. 8 B. 10
  • 23.     C. 12 D. 14 Answer: Option D Explanation: 42. The figure may be labelled as shown. The simplest triangles are AHG, AIG, AIB, JFE, CJE and CED i.e. 6 in number. The triangles composed of two components each are ABG, CFE, ACJ and EGI i.e. 4 in number. The triangles composed of three components each are ACE, AGE and CFD i.e. 3 in number. There is only one triangle i.e. AHE composed of four components. Therefore, There are 6 + 4 + 3 + 1 = 14 triangles in the given figure. 42. Find the minimum number of straight lines required to make the given figure. A. 13 B. 15 C. 17 D. 19 Answer: Option A Explanation:
  • 24.     The figure may be labelled as shown. The horizontal lines are IJ, AB, EF, MN, HG, DC and LK i.e. 7 in number. The vertical lines are AD, EH, IL, FG, BC and JK i.e. 6 in number. Thus, there are 7 + 6 = 13 straight lines in the figure. 43. Find the number of triangles in the given figure. A. 4 B. 5 C. 6 D. 7 Answer: Option B Explanation: The figure may be labelled as shown. The simplest triangles are ADE, BDF, DEF and EFC i.e. 4 in number.
  • 25.     There is only one triangle ABC composed of four components. Thus, there are 4+1 = 5 triangles in the given figure. Mirror Images VERBAL REASONING Series Completion 44. 120, 99, 80, 63, 48, ? A. 35 B. 38 C. 39 D. 40 Answer: Option A Explanation: The pattern is - 21, - 19, - 17, - 15,..... So, missing term = 48 - 13 = 35. 45. 3, 10, 101,? A. 10101 B. 10201 C. 10202 D. 11012 Answer: Option C Explanation: Each term in the series is obtained by adding 1 to the square of the preceding term. So, missing term = (101)2 + 1 = 10202. 46. 125,80,45,20,?
  • 26.     A. 5 B. 8 C. 10 D. 12 Answer: Option A Explanation: The pattern is - 45, - 35, - 25, ..... So, missing term = 20 - 15 = 5. Seating Arrangement 47. A, P, R, X, S and Z are sitting in a row. S and Z are in the centre. A and P are at the ends. R is sitting to the left of A. Who is to the right of P ? A. A B. X C. S D. Z Answer: Option B Explanation: The seating arrangement is as follows: Therefore, right of P is X. 48. A, B, C, D and E are sitting on a bench. A is sitting next to B, C is sitting next to D, D is not sitting with E who is on the left end of the bench. C is on the second position from the right. A is to the right of B and E. A and C are sitting together. In which position A is sitting ? A. Between B and D B. Between B and C C. Between E and D D. Between C and E
  • 27.     Answer: Option B Explanation: Therefore, A is sitting in between B and C. 49. Which one is immediate right to the V ? A. P B. U C. R D. T Answer: Option D Explanation: T is immediate right to the V. 50. Who is sitting immediate right to Reeta ? A. Bindu B. Rani C. Mary D. Seema Answer: Option C Explanation:
  • 28.     Mary is sitting immediate right to Reeta. DATA INT 51. Study the following table and answer the questions based on it. Expenditures of a Company (in Lakh Rupees) per Annum Over the given Years. Year Item of Expenditure Salary Fuel and Transport Bonus Interest on Loans Taxes 1998 288 98 3.00 23.4 83 1999 342 112 2.52 32.5 108 2000 324 101 3.84 41.6 74 2001 336 133 3.68 36.4 88 2002 420 142 3.96 49.4 98 1. What is the average amount of interest per year which the company had to pay during this period? A. Rs. 32.43 lakhs B. Rs. 33.72 lakhs C. Rs. 34.18 lakhs D. Rs. 36.66 lakhs 2. What is the average amount of interest per year which the company had to pay during this period? A. Rs. 32.43 lakhs B. Rs. 33.72 lakhs C. Rs. 34.18 lakhs D. Rs. 36.66 lakhs Answer: Option D Explanation:
  • 29.     Average amount of interest paid by the Company during the given period = Rs. 23.4 + 32.5 + 41.6 + 36.4 + 49.4 lakhs 5 = Rs. 183.3 lakhs 5 = Rs. 36.66 lakhs. 2. The total amount of bonus paid by the company during the given period is approximately what percent of the total amount of salary paid during this period? A. 0.1% B. 0.5% C. 1% D. 1.25% Answer: Option C Explanation: Required percentage = (3.00 + 2.52 + 3.84 + 3.68 + 3.96) x 100 % (288 + 342 + 324 + 336 + 420) = 17 x 100 % 1710 1%. 3. Total expenditure on all these items in 1998 was approximately what percent of the total expenditure in 2002? A. 62% B. 66% C. 69% D. 71% Answer: Option C Explanation: Required percentage = (288 + 98 + 3.00 + 23.4 + 83) x 100 %
  • 30.     (420 + 142 + 3.96 + 49.4 + 98) = 495.4 x 100 % 713.36 69.45%. 4. The total expenditure of the company over these items during the year 2000 is? A. Rs. 544.44 lakhs B. Rs. 501.11 lakhs C. Rs. 446.46 lakhs D. Rs. 478.87 lakhs Answer: Option A Explanation: Total expenditure of the Company during 2000 = Rs. (324 + 101 + 3.84 + 41.6 + 74) lakhs = Rs. 544.44 lakhs. 5. The ratio between the total expenditure on Taxes for all the years and the total expenditure on Fuel and Transport for all the years respectively is approximately? A. 4:7 B. 10:13 C. 15:18 D. 5:8 Answer: Option B Explanation: Required ratio = (83 + 108 + 74 + 88 + 98) (98 + 112 + 101 + 133 + 142) = 451 586
  • 31.     = 1 1.3 = 10 . 13 52. Study the following table and answer the questions. Number of Candidates Appeared and Qualified in a Competitive Examination from Different States Over the Years. State Year 1997 1998 1999 2000 2001 App. Qual. App. Qual. App. Qual. App. Qual. App. Qual. M 5200 720 8500 980 7400 850 6800 775 9500 1125 N 7500 840 9200 1050 8450 920 9200 980 8800 1020 P 6400 780 8800 1020 7800 890 8750 1010 9750 1250 Q 8100 950 9500 1240 8700 980 9700 1200 8950 995 R 7800 870 7600 940 9800 1350 7600 945 7990 885 Total number of candidates qualified from all the states together in 1997 is approximately what percentage of the total number of candidates qualified from all the states together in 1998? A. 72% B. 77% C. 80% D. 83% Answer: Option C Explanation: Required percentage = (720 + 840 + 780 + 950 + 870) x 100 % (980 + 1050 + 1020 + 1240 + 940)
  • 32.     = 4160 x 100 % 5230 = 79.54% 80%. What is the average candidates who appeared from State Q during the given years? A. 8700 B. 8760 C. 8990 D. 8920 Answer: Option C Explanation: Required average = 8100 + 9500 + 8700 + 9700 + 8950 5 = 44950 5 = 8990. In which of the given years the number of candidates appeared from State P has maximum percentage of qualified candidates? A. 1997 B. 1998 C. 1999 D. 2001 Answer: Option D Explanation: The percentages of candidates qualified to candidates appeared from State P during different years are: For 1997 780 x 100 % = 12.19%. 6400 For 1998 1020 x 100 % = 11.59%. 8800 For 1999 890 x 100 % = 11.41%. 7800
  • 33.     For 2000 1010 x 100 % = 11.54%. 8750 For 2001 1250 x 100 % = 12.82%. 9750 Maximum percentage is for the year 2001. What is the percentage of candidates qualified from State N for all the years together, over the candidates appeared from State N during all the years together? A. 12.36% B. 12.16% C. 11.47% D. 11.15% Answer: Option D Explanation: Required percentage = (840 + 1050 + 920 + 980 + 1020) x 100 % (7500 + 9200 + 8450 + 9200 + 8800) = 4810 x 100 % 43150 = 11.15% The percentage of total number of qualified candidates to the total number of appeared candidates among all the five states in 1999 is? A. 11.49% B. 11.84% C. 12.21% D. 12.57% Answer: Option B Explanation: Required percentage = (850 + 920 + 890 + 980 + 1350) x 100 % (7400 + 8450 + 7800 + 8700 + 9800) = 4990 x 100 % 42150
  • 34.     = 11.84%. 53. The following table gives the percentage of marks obtained by seven students in six different subjects in an examination. The Numbers in the Brackets give the Maximum Marks in Each Subject. Student Subject (Max. Marks) Maths Chemistry Physics Geography History Computer Science (150) (130) (120) (100) (60) (40) Ayush 90 50 90 60 70 80 Aman 100 80 80 40 80 70 Sajal 90 60 70 70 90 70 Rohit 80 65 80 80 60 60 Muskan 80 65 85 95 50 90 Tanvi 70 75 65 85 40 60 Tarun 65 35 50 77 80 80 What are the average marks obtained by all the seven students in Physics? (rounded off to two digit after decimal) A. 77.26 B. 89.14 C. 91.37 D. 96.11 Answer: Option B Explanation: Average marks obtained in Physics by all the seven students = 1 x [ (90% of 120) + (80% of 120) + (70% of 120) 7 + (80% of 120) + (85% of 120) + (65% of 120) + (50% of 120) ] = 1 x [ (90 + 80 + 70 + 80 + 85 + 65 + 50)% of 120 ]
  • 35.     7 = 1 x [ 520% of 120 ] 7 = 624 7 = 89.14. The number of students who obtained 60% and above marks in all subjects is? A. 1 B. 2 C. 3 D. None Answer: Option B Explanation: From the table it is clear that Sajal and Rohit have 60% or more marks in each of the six subjects. What was the aggregate of marks obtained by Sajal in all the six subjects? A. 409 B. 419 C. 429 D. 449 Answer: Option D Explanation: Aggregate marks obtained by Sajal = [ (90% of 150) + (60% of 130) + (70% of 120) + (70% of 100) + (90% of 60) + (70% of 40) ] = [ 135 + 78 + 84 + 70 + 54 + 28 ] = 449. In which subject is the overall percentage the best?
  • 36.     A. Maths B. Chemistry C. Physics D. History Answer: Option A Explanation: We shall find the overall percentage (for all the seven students) with respect to each subject. The overall percentage for any subject is equal to the average of percentages obtained by all the seven students since the maximum marks for any subject is the same for all the students. Therefore, overall percentage for: (i) Maths = 1 x (90 + 100 + 90 + 80 + 80 + 70 + 65) % 7 = 1 x (575) % 7 = 82.14%. (ii) Chemistry = 1 x (50 + 80 + 60 + 65 + 65 + 75 + 35) % 7 = 1 x (430) % 7 = 61.43%. (iii) Physics = 1 x (90 + 80 + 70 + 80 + 85 + 65 + 50) % 7 = 1 x (520) % 7 = 74.29%. (iv) Geography = 1 x (60 + 40 + 70 + 80 + 95 + 85 + 77) % 7
  • 37.     = 1 x (507) % 7 = 72.43%. (v) History = 1 x (70 + 80 + 90 + 60 + 50 + 40 + 80) % 7 = 1 x (470) % 7 = 67.14%. (vi) Comp. Science = 1 x (80 + 70 + 70 + 60 + 90 + 60 + 80) % 7 = 1 x (510) % 7 = 72.86%. Clearly, this percentage is highest for Maths. What is the overall percentage of Tarun? A. 52.5% B. 55% C. 60% D. 63% Answer: Option C Explanation: Aggregate marks obtained by Tarun = [ (65% of 150) + (35% of 130) + (50% of 120) + ((77% of 100) + (80% of 60) + (80% of 40) ] = [ 97.5 + 45.5 + 60 + 77 + 48 + 32 ] = 360.
  • 38.     The maximum marks (of all the six subjects) = (150 + 130 + 120 + 100 + 60 + 40) = 600. Overall percentage of Tarun = 360 x 100 % = 60%. 600 PERCENTAGE: 54. A student multiplied a number by 3 instead of 5 . 5 3 What is the percentage error in the calculation? A. 34% B. 44% C. 54% D. 64% Answer: Option D Explanation: Let the number be x. Then, error = 5 x - 3 x = 16 x. 3 5 15 Error% = 16x x 3 x 100 % = 64%.15 5x 55. Two tailors X and Y are paid a total of Rs. 550 per week by their employer. If X is paid 120 percent of the sum paid to Y, how much is Y paid per week? A. Rs. 200 B. Rs. 250 C. Rs. 300 D. None of these Answer: Option B
  • 39.     Explanation: Let the sum paid to Y per week be Rs. z. Then, z + 120% of z = 550. z + 120 z = 550 100 11 z = 550 5 z = 550 x 5 = 250.11 56. Gauri went to the stationers and bought things worth Rs. 25, out of which 30 paise went on sales tax on taxable purchases. If the tax rate was 6%, then what was the cost of the tax free items? A. Rs. 15 B. Rs. 15.70 C. Rs. 19.70 D. Rs. 20 Answer: Option C Explanation: Let the amount taxable purchases be Rs. x. Then, 6% of x = 30 100 x = 30 x 100 = 5.100 6 Cost of tax free items = Rs. [25 - (5 + 0.30)] = Rs. 19.70 57. Rajeev buys good worth Rs. 6650. He gets a rebate of 6% on it. After getting the rebate, he pays sales tax @ 10%. Find the amount he will have to pay for the goods. A. Rs. 6876.10 B. Rs. 6999.20 C. Rs. 6654 D. Rs. 7000 Answer: Option A Explanation:
  • 40.     Rebate = 6% of Rs. 6650 = Rs. 6 x 6650 = Rs. 399. 100 Sales tax = 10% of Rs. (6650 - 399) = Rs. 10 x 6251 = Rs. 625.10 100 Final amount = Rs. (6251 + 625.10) = Rs. 6876.10 ---------------------- 58. The ages of the two persons differ by 20 years. If 5 year ago, the older one be 5 times as old as the younger one, then their present ages, in year are: A. 25, 5, B. 30, 10 C. 35, 15 D. 50, 30 59. In what ratio must rice at Rs 9.30 per Kg be mixed with rice at Rs 10.80 per Kg so that the mixture be worth Rs 10 per Kg? A. 6:5 B. 8:7 C. 3:7 D. 6:1 Answer – (B) Solution: C.P of 1 Kg C.P of 1 Kg rice of 1st rice of 2nd kind (930p) kind (1080p) / Mean Price (1000p) / (1080 – 1000) : (1000 - 930) 80 70 Thus, required ratio = 80 : 70 = 8 : 7
  • 41.     60. How much water must be added to 60 litres of milk at 1.5 litres for Rs. 20 So as to have a mixture worth Rs.10 2/3 a litre? A. 10 litres B. 12 litres C. 15 litres ✖ D. 18 litres Answer – (C) Solution: C.P. of 1 litre of milk =Rs. (20x2) 3 =Rs. 40 3 C.P. of 1 litre of water = 0 Mean price = Rs. 32 3 By the rule of alligation, we have: C.P of 1 litre C.P of 1 litre Of water of milk (0) (Rs. 40/3) / Mean Price (Rs. 32/3) / ( 40 3 –
  • 42.     32 3 ) ( 32 3−0 ) 8/3 32/3 Ratio of water and milk = 8 3 : 32 3 =8:32=1:4 Thus, Quantity of water to be added to 60 litres of milk = ( 1 4 ×60)litres= 15 litres 61. In what ratio must wheat at Rs.3.20 per kg be mixed with wheat at Rs.2.90 per kg so that the mixture be worth Rs.3.08 per kg? A. 3 : 4 B. 2 : 3 C. 3 : 2 ✖ D. 4 : 3 Answer – (C) Solution: C.P of a unit quantity of 1st kind = Rs. 3.20 C.P of a unit quantity of 2nd kind = Rs. 2.90 Mean price = Rs.3.08 C.P of unit quantity C.P of unit quantity of 1st kind of 2nd kind (Rs. 3.20) (Rs. 2.90)
  • 43.     / Mean Price (Rs.3.08) / (3.08 – 2.90) : (3.20 – 3.08) 0.18 0.12 Required ratio = 0.18 : 0.12 = 3 : 2 62. In what proportion must rice at Rs. 3.10 per kg be mixed with rice at Rs. 3.60 per kg so that the mixture be worth Rs. 3.25 per kg? A. 3 : 7 B. 5 : 3 C. 3 : 5 ✔ D. 7 : 3 Answer – (D) Solution: C.P of a unit quantity of 1st kind = 310p (in paise) C.P of a unit quantity of 2nd kind = 360p Mean price = 325p C.P of unit quantity C.P of unit quantity of 1st kind of 2nd kind (310p) (360p) / Mean Price (325p) / (360 – 325) : (325 – 310) 35 15 Required ratio = 35 : 15 = 7 : 3 They must be mixed in the ratio 7 : 3 AVERAGE:
  • 44.     68. The average of first five multiples of 3 is : A. 9 B. 10 C. 8 ✖ D. 11 Answer – (A) Solution: Basic Formula: 1, 2, 3 ..… n If n is odd, the formula is ( n+1 2 )th term The five multiples of 3 is 3, 6, 9, 12, 15 ( n+1 2 ) ⇒ ( 5+1 2 )th term ⇒ 6 2 th term =3rd term Here 3rd term is 9 69. There are two sections A and B of a class, consisting of 36 and 44 students’ respectively. If the average weight of section A is 40kg and that of section B is 35kg, find the average weight of the whole class. A. 30 kg B. 35 kg C. 42.5 kg D. 37.25 kg
  • 45.     Answer – (D) Solution: Total weight of (36+44) Students = (36x40+44x35)kg = 2980kg Therefore average weight of the whole class = (2980/80) kg Therefore average weight = 37.25kg Distance between two stations A and B is 778km. A train covers the journey from A to B at 84km per hour and returns back to A with a uniform speed of 56km per hour. Find the average speed of train during the whole journey. A. 60 km/hr B. 30.5 km/hr C. 57 km/hr ✔ D. 67.2 km/hr Answer - (D) Solution: Average speed=( 2xy x+y )km/hr =( 2×84×56 84+56 ) km/hr =( 2×84×56 140 )km/hr = 67.2km/hr. APTITUDE: Exam2win > Aptitude Question Papers (All Topics)
  • 46.     APTITUDE QUESTIONS AND ANSWERS 1 to 10 70. 1. A man lends Rs. 10000 in four parts. If be gets 8% on Rs. 2000 7 on Rs. 4000 and 8 on Rs. 1400 what per cent must he get for the remainder, if the average interest is 8.13%? 7% 9% 9 10 ANSWER : 9% 71. On july 2, 1985 it was Wednesday. The day of the week on July 2, 1984 was: Wednesday Tuesday Monday Thursday ANSWER : Monday 72. A tank can be filled by one tap in 20 min, and by another in 25 min. both the taps are kept open for 5 min, and then the second is turned off. In how many minutes more is the tank completely filled? 17 min 12 min 11 min 6 min ANSWER : 1 min
  • 47.     73. If each side of a cube is doubled, then its volume : is doubled becomes 4 times becomes 6 times becomes 8 times ANSWER : becomes 8 times 74. A sum of money is sufficient to pay A’s wages for 21 days or B’s wages for 28 days. The money is sufficient to pay the wages of both for: 12 days 12 days 14 days none of these ANSWER : 12 days 75. The mean proportional between 0.32 and 0.02 is: 0.34 0.3 0.16 0.08 ANSWER : .08 76. The ratio of milk and water in 66 kg. of adulterated milk is 5 : 1. Water is added to it to make the ratio 5 : 3. The quantity of water added is : 22 kg. 24.750 kg.
  • 48.     16.500 kg. 20 kg. ANSWER : 22 kg. 77. The length of hall is 4/3) times its breadth. If the area of the hall be 300 square metres the difference between the length and the breadth is: 15 metres 4 metres 3 metres none of these ANSWER : none of these 78. A can run a kilometer in 4 min. 54 sec. and B in 5 min. How many metres start can A give B in a km race so that the race may end in a dead heat? 20 metres 16 metres 18 metres 14.5 metres ANSWER : 20 metres 79. A,B,C, enter into a partnership and their capitals are in the proportion of . A withdraws half his capital at the end of 4 months. Out of a total annual profit of Rs. 847, A’ share is: Rs. 252 Rs. 280 Rs.315 Rs. 412 ANSWER : Rs. 280
  • 49.     AVERAGE APTITUDE QUESTIONS AND ANSWERS 21 to 30 80. The average age of an adult class is 40 years. 12 new students with an average age of 32 years join the class, thereby decreasing the average by 4 years. The original strength of the class was : 10 11 12 15 ANSWER : 12 81. A cistern is normally filled in 8 hours but takes two hours longer to fill because of a leak in it sbottom. If the cistern is full, the leak will empty it in: 16 hrs 20 hrs 25 hrs 40 hrs ANSWER : 0 hrs 82. The difference between simple interest and compound interest on a sum for 2 years at 8% when the interest is compounded annually is Rs.16. If the interest were compounded half yearly, the difference in two interests would be nearly: Rs.16 Rs.16.80 Rs. 21.85 RS.24.64 ANSWER : RS.24.64 83. At what time between 9 and 10 will the hands of a watch be together ? 45 minutes past 9 50 minutes past 9 49 minutes past 9
  • 50.     48 minutes past 9 ANSWER : 49 minutes past 9 84. If I purchased 11 books for Rs. 10 and sold all the books at the rate of 10 books for Rs. 11, the profit per cent is: 10% 11% 21% 100% ANSWER : 21% 85. A fraction becomes 4 when 1 is added to the both the numerator and denominator; and it becomes 7 when 1 is subtracted from both the numerator and denominator. The numerator of the given fraction is: 2 3 7 15 ANSWER : 15 85. The current of a stream runs at 1 km/hr. A motor boat goes 35 km upstream and back again to the starting point in 12 hours. The speed of motorboat in still water is : 6 km/hr 7 km/hr 8.5 km/hr 8 km/hr ANSWER : 6 km/hr 86. A man sold 20 articles for Rs. 60 and gained 20%. How many articles did he buy for Rs.60? 22 24 25 26 ANSWER : 24
  • 51.     87. X an Y are two stations 500 km apart. A train starts form X and moves towards Y at 20 km/hr. Another rain starts from Y at the same time and moves towards X at 30 km/hr. How far from X will they cross each other: 200 km 300 km 120 km 40 km ANSWER : 200 km 88. P.V. Narsimha Rao was elected party leader on 29 th may. 1991. What was the day of the week? Tuesday Friday Wednesday Sunday ANSWER : Wednesday AVERAGE APTITUDE QUESTIONS AND ANSWERS 21 to 30 89. The average age of an adult class is 40 years. 12 new students with an average age of 32 years join the class, thereby decreasing the average by 4 years. The original strength of the class was : 10 11 12 15 ANSWER : 12 90. A cistern is normally filled in 8 hours but takes two hours longer to fill because of a leak in it sbottom. If the cistern is full, the leak will empty it in: 16 hrs 20 hrs 25 hrs
  • 52.     40 hrs ANSWER : 0 hrs 91. The difference between simple interest and compound interest on a sum for 2 years at 8% when the interest is compounded annually is Rs.16. If the interest were compounded half yearly, the difference in two interests would be nearly: Rs.16 Rs.16.80 Rs. 21.85 RS.24.64 ANSWER : RS.24.64 92. At what time between 9 and 10 will the hands of a watch be together ? 45 minutes past 9 50 minutes past 9 49 minutes past 9 48 minutes past 9 ANSWER : 49 minutes past 9 93. If I purchased 11 books for Rs. 10 and sold all the books at the rate of 10 books for Rs. 11, the profit per cent is: 10% 11% 21% 100% ANSWER : 21% 94. A fraction becomes 4 when 1 is added to the both the numerator and denominator; and it becomes 7 when 1 is subtracted from both the numerator and denominator. The numerator of the given fraction is: 2 3 7 15
  • 53.     ANSWER : 15 95. The current of a stream runs at 1 km/hr. A motor boat goes 35 km upstream and back again to the starting point in 12 hours. The speed of motorboat in still water is : 6 km/hr 7 km/hr 8.5 km/hr 8 km/hr ANSWER : 6 km/hr 96. A man sold 20 articles for Rs. 60 and gained 20%. How many articles did he buy for Rs.60? 22 24 25 26 ANSWER : 24 97. X an Y are two stations 500 km apart. A train starts form X and moves towards Y at 20 km/hr. Another rain starts from Y at the same time and moves towards X at 30 km/hr. How far from X will they cross each other: 200 km 300 km 120 km 40 km ANSWER : 200 km 98. P.V. Narsimha Rao was elected party leader on 29 th may. 1991. What was the day of the week? Tuesday Friday Wednesday Sunday ANSWER : Wednesday
  • 54.     VOLUME AND SURFACE AREA QUESTIONS AND ANSWERS 1 to 3 99. The length of diagonal of a cube is (14 x ) cm. The volume of the cube is : 2744 cm3 2744 cm3 588 cm3 3528 cm3 ANSWER : 2744 cm3 100. The surface area of a cuboid 22 cm by 12 cm by 7.5 cm, is : 1980 cm2 2076 cm2 1038 cm2 none of these ANSWER : 1038 cm2 101. Given that 1 cubic cm of marble weight 25 gms, the weight of a marble block 28 cm in width and 5 cm thick is 112 kg. the length of the block is : 36 cm 37.5 cm 32 cm 26.5 cm ANSWER : 32 cm PROBLEMS ON NUMBERS QUESTIONS AND ANSWERS 1 to 3 102. If one-fourth of one-third of one-half of a number is 15, the number is: 72 120 180 360 ANSWER : 360
  • 55.     103. Three numbers are in the ratio 3:4:5. The sum of the largest and the smallest equals the sum of the third and 52. The smallest number is: 20 27 39 52 ANSWER : 39 104. The sum of three consecutive odd numbers is 57. The middle one is: 19 21 23 17 ANSWER : 19 PARTNERSHIP QUESTIONS AND ANSWERS 1 to 3 105. Jayant started a business, investing Rs. 6000. Six months later madhu joined him, investing Rs. 4000, If they made a profit of Rs. 5200 at the end of the year, how much must be the share of Madhu? Rs. 2080 Rs.1300 Rs.1800 Rs. 2600 ANSWER : Rs.1800 106. A and B invest in business in the ratio 3 :2. If the 5% of the total profit goes to charity and A?s share is Rs. 855, total profit is: Rs.1576 Rs. 1537.50 Rs.1500 Rs.1425 ANSWER : Rs. 1500
  • 56.     107. A, B and c invest Rs. 2000, Rs.3000 and Rs. 4000 in a business, After one year, A removed his money but B and C continued for one more year. If the net profit after 2 years be Rs.3200, them A?s share in the Profit is: Rs. 1000 Rs. 600 Rs. 800 Rs. 400 ANSWER : Rs. 400 108. Which of the following diagrams indicates the best relation between Travelers, Train and Bus ? A. B. C. D. Answer & Explanation Answer: Option C Explanation: Bus and Train are different from each other but some travelers travel by bus and some travel by train. 109. Which of the following diagrams indicates the best relation between Profit, Dividend and Bonus ? A. B. C. D. Answer & Explanation Answer: Option B Explanation: Bonus and Dividend are different from each other. But both these are parts of profit
  • 57.     110. Which of the following diagrams indicates the best relation between Women, Mothers and Engineers ? A. B. C. D. Answer & Explanation Answer: Option A Explanation: All mothers are women and some mothers and some women may be engineers. 111. Which of the following diagrams indicates the best relation between Factory, Product and Machinery ? A. B. C. D. Answer & Explanation Answer: Option D Explanation: Product and Machinery are different from each other but both are found in Factory. 112. Which of the following diagrams indicates the best relation between Author, Lawyer and Singer ? A. B. C. D. Answer & Explanation Answer: Option B
  • 58.     Explanation: All the three are different professions. 113. Which of the following diagrams indicates the best relation between Judge, Thieves and Criminals ? A. B. C. D. Answer & Explanation Answer: Option B Explanation: All the thieves are criminals while judge is different from these. 114. Which of the following diagrams indicates the best relation between Judge, Thieves and Criminals ? A. B. C. D. 116. Which of the following diagrams indicates the best relation between Pigeon, Bird and Dog ? A. B. C. D. Answer & Explanation Answer: Option A Explanation: All the pigeons are birds while dog is different from these.
  • 59.     117. Which of the following diagrams indicates the best relation between Earth, Sea and Sun ? A. B. C. D. Answer & Explanation Answer: Option A Explanation: Sea is a part of Earth while Sun is different from these two. 118. Which of the following diagrams indicates the best relation between Hockey, Football and Cricket ? A. B. C. D. Answer & Explanation Answer: Option B Explanation: All these three games are different from each other. 119. One morning Udai and Vishal were talking to each other face to face at a crossing. If Vishal's shadow was exactly to the left of Udai, which direction was Udai facing? A. East B. West C. North D. South Answer & Explanation Answer: Option C Explanation:
  • 60.     120. Y is in the East of X which is in the North of Z. If P is in the South of Z, then in which direction of Y, is P? A. North B. South C. South-East D. None of these Answer & Explanation Answer: Option D Explanation: P is in South-West of Y. 121. If South-East becomes North, North-East becomes West and so on. What will West become? A. North-East B. North-West C. South-East D. South-West Answer & Explanation Answer: Option C Explanation: It is clear from the diagrams that new name of West will become South-East.
  • 61.     121. A man walks 5 km toward south and then turns to the right. After walking 3 km he turns to the left and walks 5 km. Now in which direction is he from the starting place? A. West B. South C. North-East D. South-West Answer: Option D Explanation: Hence required direction is South-West. 123. Rahul put his timepiece on the table in such a way that at 6 P.M. hour hand points to North. In which direction the minute hand will point at 9.15 P.M. ? A. South-East B. South C. North D. West Answer & Explanation Answer: Option D Explanation: At 9.15 P.M., the minute hand will point towards west.
  • 62.     124. Dev, Kumar, Nilesh, Ankur and Pintu are standing facing to the North in a playground such as given below: 1. Kumar is at 40 m to the right of Ankur. 2. Dev is are 60 m in the south of Kumar. 3. Nilesh is at a distance of 25 m in the west of Ankur. 4. Pintu is at a distance of 90 m in the North of Dev. 1. Which one is in the North-East of the person who is to the left of Kumar? A. Dev B. Nilesh C. Ankur D. Pintu Answer & Explanation Answer: Option D Explanation: Ankur is in the left of Kumar. Hence Pintu is in North-East of Ankur. 124. Each of the following questions is based on the following information: 1. Six flats on a floor in two rows facing North and South are allotted to P, Q, R, S, T and U. 2. Q gets a North facing flat and is not next to S. 3. S and U get diagonally opposite flats. 4. R next to U, gets a south facing flat and T gets North facing flat. 1. If the flats of P and T are interchanged then whose flat will be next to that of U? A. P B. Q C. R D. T Answer & Explanation
  • 63.     Answer: Option C Explanation: Hence flat R will be next to U. 126. Rasik walked 20 m towards north. Then he turned right and walks 30 m. Then he turns right and walks 35 m. Then he turns left and walks 15 m. Finally he turns left and walks 15 m. In which direction and how many metres is he from the starting position? A. 15 m West B. 30 m East C. 30 m West D. 45 m East Answer & Explanation Answer: Option D Explanation:
  • 64.     127. Solution: Hank: pear apple cherry rose Sam: cherry onion rose tulip Paul: carrot gourd onion rose Zick: aster rose tulip lily Luke: pear nut gourd parsley 128. Solution: The Spanish ship goes to Port Said and the French ship carries tea. However, tea can be carried by the Brazilian ship, too. If you understood position 'to the right' to mean anywhere on the right side from the given point (not only right next to). French 5:00 tea blue Genoa Greek 6:00 coffee red Hamburg Brazilian 8:00 cocoa black Manila English 9:00 rice white Marseille Spanish 7:00 corn green Port Said
  • 65.     129. Solution: 1 2 3 4 5 Name Bob Rachael Eilish Keeley Amy Favorite TV programme The Simpsons Coronation Street Desperate Housewives Neighbours Eastenders Destination France Italy Africa England Australia Age 46 21 81 52 14 Where they live Town Youth Hostel Farm Village City Hairstyle Bald Long Afro Curly Straight 130. Solution: Holly did! See below for full solution: Person Alex Jackie Julieanne Cameron Holly Flavor Vanilla Strawberry Banana Caramel Chocolate Cookie Jar Mini Brass Tall Square Round Brand Coles Dick Smith Paradise No Frills Arnotts Cookie Choc chip Tiny Teddies 100s and 1000s Oreos Ginger Place Lounge Bedroom Kitchen Closet Table 131. Solution: Betty, Carol and Dan work in Administration department. Earl works in the Marketing department. 132. house 1 2 3 4 5 color yellow blue red green white nationality norweigian dane brit german swede drink water tea milk coffee beer
  • 66.     smoke dunhill blend pall mall prince bluemaster pet cats horse birds fish dogs 134. Solution  888 + 88 + 8 + 8 + 8  (8(8(8+8)-(8+8)/8))-8  8888/8.888  (888-8) + 8×(8+8) – 8  ((8×(8+8))-((8+8+8)/8))*8  (8+((8+8)/8))^((8+8+8)/8)  (8+((8+8)/8))^((88/8)-8)  ((8×(8+8))-((88/8)-8))×8  (8888-888)/8  8(8×8+8×8)-8-8-8 135. Solution: Tom's Granny is 58 years old. Let's see why: First child born: Granny is 19 Second child born: Granny is 23 (19 + 4) Third child born: Granny is 27 (23 + 4) Fourth child born: Granny is 31 (27 + 4) Fifth child born: Granny is 35 (31 + 4) Sixth child born: Granny is 39 (35 + 4) Sixth child is 19: Granny is 58 (39 + 19) 136. Solution: The left number multiplied by 5 gives the total of the other two: 27 × 5 = 56 + 79 23 × 5 = 88 + 27 16 × 5 = 56 + ?? So ?? = 16 × 5 - 56 = 80 - 56 = 24 137. Solution:
  • 67.     The product of the two largest minus the square of the smallest, So the missing number is 45-16 = 29 138. 139. Solution: Divide the 9 balls into 3 groups of 3. Compare the weight of two of those groups. The heavier group should then be obvious, it will either tip the scales, or, if the scales stay balanced, then it is the group you didn't include. Now, choose 2 balls from this group and compare their weights, and using the same logic as before, the heavier ball will be obvious. 140. Solution: One possible method follows: 1. Fill up the 5 litre bucket 2. Pour the 5 litre bucket into the 3 litre bucket until the 3 litre bucket is full.
  • 68.     3. Empty the 3 litre bucket. 4. Pour the remaining 2 litres from the 5 litre bucket into the 3 litre bucket. 5. Fill up the 5 litre bucket again.